CLIPP cases - End of case Questions

अब Quizwiz के साथ अपने होमवर्क और परीक्षाओं को एस करें!

A 33-year-old G1P0 female with a history of medically controlled seizures gives birth vaginally to a boy with IUGR at 38 weeks' gestation. The newborn is noted to have dysmorphic cranial features and his head circumference is 28.5 cm (< 5th percentile). What is another associated abnormality you might expect to see in this newborn? A. Hepatosplenomegaly B. Cardiac defects C. Absent red reflex D. Chorioretinitis E. Tremors

Q1.1 > B has been selected by the expert. A. Hepatosplenomegaly in newborns is seen in metabolic diseases, storage diseases, HIV vertical transmission, intrinsic liver disease, and in congenital infections. Maternal anticonvulsant use does not cause hepatosplenomegaly. B. The mother was on an anticonvulsant for her seizures. Taking anticonvulsants during pregnancy may lead to cardiac defects, dysmorphic craniofacial features, hypoplastic nails and distal phalanges, IUGR, and microcephaly. Mental retardation may be seen. A rare neonatal side effect is methemoglobinuria. C. The red reflex is the normal reddish-orange reflection of light from the eye's retina that is observed when using an ophthalmoscope. An absent red reflex in a newborn can be due to congenital cataracts or a retinoblastoma. D. Chorioretinitis in a newborn may be due to congenital toxoplasmosis and CMV infections. It is not caused by maternal anticonvulsant use. E. Tremors in a newborn can be due to maternal substance use. They would not result from maternal use of anticonvulsants.

A 19-year-old female in her 38th week of pregnancy goes into active labor. Shortly after birth her baby is noted to have a high-pitched cry, tremulousness, hypertonicity, and feeding difficulties. The baby is otherwise developmentally normal and the remainder of the physical exam also is normal. What is the drug the baby's mother likely used during her pregnancy? A. Heroin B. Alcohol C. Marijuana D. Cocaine E. Tobacco

Q1.2 > A has been selected by the expert. A. Heroin is the correct choice. Opiate use during pregnancy may result in several different symptoms, including CNS findings (irritability, hyperactivity, hypertonicity, incessant high-pitched cry, tremors, seizures), GI symptoms (vomiting, diarrhea, weight loss, poor feeding, incessant hunger, excessive salivation), and respiratory findings (including nasal stuffiness, sneezing, and yawning). B. Alcohol is incorrect. Fetal alcohol syndrome has a distinct pattern of facial abnormalities, growth deficiency, and CNS dysfunction. These infants may also exhibit other neurobehavioral deficits such as poor motor skills and hand-eye coordination and learning problems, such as difficulties with memory, attention, and judgment. C. Marijuana is incorrect. There is limited evidence for a withdrawal syndrome associated with marijuana use. D. Cocaine use during pregnancy is not typically associated with withdrawal symptoms. Cocaine has been linked to subtle deficits appreciated later in childhood, including deficits in cognitive performance, information processing, and attention to tasks. E. Tobacco is incorrect. Smoking is not associated with the withdrawal syndrome described above. Smoking has been linked in a dose-dependent manner with lower weight newborns at birth. There is a two-fold increase in low birth weight even in light smokers (< 10 cigarettes per day). Smoking during pregnancy also has been associated with subtle neurodevelopmental deficits in some exposed children.

A 19-year-old G1P0 presents in labor to the ED at 38 gestational weeks. On interview it is discovered that the patient had irregular prenatal care, drank a couple of beers every weekend, and smoked 4 cigarettes a day. She delivers a baby boy who is small for gestational age. On exam, it is noted the baby has microcephaly, a smooth philtrum, and a thin upper lip. What do you suspect caused these features in the baby? A. Tobacco exposure B. Alcohol exposure C. Congenital rubella D. Vertically transmitted HIV E. Congenital CMV infection

Q1.3 > B has been selected by the expert. A. This choice is incorrect. While tobacco exposure can cause infants to be small for gestational age they typically do not have any characteristic facial features. B. This choice is correct. Fetal alcohol syndrome has very characteristic facial features, including a smooth philtrum, thinning of the upper lip, and small palpebral fissures. C. This choice is incorrect. Congenital rubella presents with sensorineural deafness, eye abnormalities (retinopathy, cataracts), and patent ductus arteriosus. D. This choice is incorrect. Typically, vertically transmitted HIV does not lead to recognizable symptoms at birth. This diagnosis cannot be completely ruled out without lab testing. E. This choice is incorrect. Symptomatic congenital CMV infection presents with microcephaly, jaundice, hepatosplenomegaly, low birth weight, and petechiae at birth.

A mother brings her 20-day-old male infant to your clinic for the child's first visit. You learn that the infant was born at home to a 28-year-old G1P1, and the infant has not yet received newborn screening. During your history, you learn that the infant has been vomiting 2 to 3 times per day, and the mother reports that her son seems fussier than her friends' infants. On exam, you note an eczematous rash and a musty odor to the infant's skin and urine. Which enzyme deficiency would you expect the infant to display? A. Phenylalanine hydroxylase B. Cystathionine synthase C. Sphingomyelinase D. Alpha-L-iduronidase E. Glucose-6-phosphatase

Q1.4 > A has been selected by the expert. A. This infant likely has phenylketonuria (PKU), an autosomal recessive disorder of amino acid metabolism caused by a deficiency in the enzyme phenylalanine hydroxylase. Affected infants are normally detected by newborn screening, but can present with vomiting, hypotonia, musty odor, developmental delay, and decreased pigmentation of the hair and eyes. The best developmental outcomes occur if a phenylalanine-restricted diet is initiated in infancy. B. A defect in cystathionine synthase occurs in homocystinuria, a disorder of amino acid metabolism. Homocystinuria is inherited in an autosomal recessive pattern. Individuals display Marfanoid body habitus, a hypercoaguable state, and possible developmental delay. The condition can be diagnosed by testing for increased methionine in a patient's urine or blood. C. A defect in sphingomyelinase occurs in Niemann-Pick disease, a lysosomal storage disease. Children present by six months of age with hepatomegaly, ataxia, seizures, and progressive neurologic degeneration. Fundoscopic exam reveals a "cherry-red" macula. D. A defect in alpha-L-iduronidase occurs in Hurler syndrome, a type of autosomal recessive lysosomal storage disease. Children typically do not display symptoms until one year of age. Symptoms include hepatosplenomegaly, coarse facial features, frontal bossing, corneal clouding, and developmental delay. Affected individuals typically do not live past fifteen years old. E. A defect in glucose-6-phosphatase occurs in Von Gierke's disease, a glycogen storage disease. Von Gierke's disease is inherited in an autosomal recessive pattern. Individuals present with hypoglycemia, hepatomegaly, and metabolic acidosis.

Johnny is a 25-month-old male who presents to the ED with a 2-day history of vomiting and diarrhea. Dad relays a history of abrupt onset of vomiting that started yesterday around 1 pm. Johnny has had 6 episodes of emesis since yesterday and 3 episodes of diarrhea. The emesis is non-bilious and the diarrhea is described as watery with specks of blood throughout the diarrhea. There are no sick contacts in the home. Vital signs: T 37.1, P 102, R 20, BP 90/60. Physical examination is normal and Johnny has still been tolerating some PO feeds without instant vomiting. What is the most immediate intervention for this patient? A. IV bolus with D5W B. IV bolus with 0.9% saline C. CT scan and surgical consult D. random glucose test E. no immediate intervention is necessary

Q15.1 > E has been selected by the expert. A. An IV bolus with D5W is indicated in cases of confirmed hypoglycemia and is used for maintenance fluids. This is not indicated in this patient. B. With his normal vitals and no obvious signs of dehydration, an IV bolus of 0.9% saline is not indicated in this patient. If there was evidence of dehydration on physical exam or with vitals, then IV fluids would be necessary. C. This vignette does not seem like a surgical case. Physical examination was normal, which makes abdominal pathology less likely. While appendicitis might be in the differential diagnosis, other diagnoses are more likely, making the excessive radiation exposure from a CT scan not necessary. D. With cases of dehydration one must always think about abnormalities in blood glucose levels. This clinical presentation does not suggest signs of hypo or hyperglycemia. E. At this point the patient is most likely suffering from a case of viral gastroenteritis. Because he is still tolerating some PO feeds, has no obvious signs of dehydration, and has normal vital signs, there is no need for aggressive IV fluid administration or diagnostic work up. Strict return precautions should be given and it should be advised that Johnny maintains fluids as much as possible.

Rashid is a 5-week-old infant who presents to clinic with 4 days of repeated, forceful, non-bilious, non-bloody vomiting without diarrhea. He has 8 to 9 episodes of vomiting per day immediately following breastfeeding. The episodes started 2 weeks after the entire family suffered from severe viral gastroenteritis. His birth history is uncomplicated (full term, NSVD, unremarkable 30-week ultrasound) and birth weight was 3.6 kg (50th percentile). On exam, his vitals are: T 36.7°C, HR 185, BP 85/45, RR 36, Wt 4.1 kg (25th percentile). On exam, his eyes are moderately sunken without production of tears, his lips are cracked, and his throat is without erythema. His capillary refill is ~3 seconds, and his pulse is thready. What is your first step in management? A. Close observation in the office for 6 hours and encourage PO intake until vitals normalize. B. Intravenous lactated Ringer's solution of 20mL/kg boluses until baseline clinical status is achieved, then 100 mL/kg oral rehydration solutions over next 4 hours. C. Intravenous 20 mL/kg boluses of ¼ normal saline solution until baseline clinical status is achieved, then closely monitor vitals for 6 hours while encouraging PO formula intake. D. Observe for 6 hours with normal PO intake and administer 60-120 mL of oral rehydration solution for every episode of vomiting. E. Administer 75 mL/kg of oral rehydration solution over 3-4 hours and 60-120 mL of oral rehydration solution for every episode of vomiting.

Q15.2 > B has been selected by the expert. A. Observing closely in the office for 6 hours and encouraging PO intake until vitals normalize are not appropriate measures for treating this child's severe dehydration. Signs of severe dehydration include lethargy or unconsciousness on exam, poor PO intake, tachycardia, weak or nonpalpable pulses, deep breathing, deeply sunken eyes, parched mouth and tongue, reduced skin turgor, and cold/cyanotic extremities. In cases like this, the child must be placed on immediate IV fluids with 20 mL/kg boluses until vitals and mental status normalize. B. Lactated Ringer's solution or normal saline in 20 mL/kg boluses until urine output is established and mental status improves, then 100 mL/kg oral rehydration solutions over next 4 hours. This follows current CDC guidelines for treating a severely dehydrated child. Intravenous hydration with 5% dextrose ½ normal saline at twice maintenance fluid rates may be substituted for the oral rehydration solution if the child is not tolerating PO intake. To replace ongoing losses, the CDC recommends 60-120mL of oral rehydration solution per diarrheal/emetic episode (through a nasogastric tube, if necessary). C. One quarter normal saline (1/4 NS) is a hypotonic solution, and would not be ideal for the treatment of dehydration due to emesis. The recommended therapy to correct severe dehydration is to give 20 mL/kg boluses of isotonic solution and to reassess for clinical improvement following each administration. Once the patient is stable and back to baseline, then continue IV hydration with 5% dextrose ½ normal saline at twice maintenance fluid rates OR give 100 mL/kg oral rehydration solution over 4 hours. D. This would not be a recommended treatment for the severely dehydrated child since it relies on normal PO intake. However, the replacement of losses strategy is correct for all patients < 10 kgs. E. IV rehydration is key to initial fluid resuscitation in the severely dehydrated, so this treatment would not be sufficient. However, this plan would be appropriate for mild to moderately dehydrated patients.

A 6-month-old infant comes to clinic because of several weeks of vomiting after large feedings. The vomiting has become blood-streaked, which is when the mom became concerned and brought him in. The baby's PO intake has been down and he has been losing weight. Abdominal exam is normal, with no masses palpated. What is the most likely diagnosis? A. Pyloric stenosis B. Gastroenteritis C. GERD D. Volvulus E. Intussusception

Q15.3 > C has been selected by the expert. A. Choice A is incorrect because pyloric stenosis is characterized by a pattern of forceful, projectile, non-bilious vomiting, usually in younger infants. Infants are usually hungry and nurse avidly. An oval mass, 5-15 mm in longest dimension can be felt on deep palpation in the right upper abdomen, especially after vomiting. B. Choice B is incorrect. Large watery stools, which this patient did not have, are the hallmark of infectious gastroenteritis. Also, gastroenteritis is a more acute disease, while this patient is having more chronic symptoms. C. Choice C is correct because regurgitation/spitting up may be difficult to distinguish from true vomiting. Infants who reflux with overfeeding may sometimes have forceful vomiting. Severe esophagitis may result in blood-streaked emesis. Pain from reflux or esophagitis may lead to feeding aversion when gastroesophageal reflux is severe. D. Choice D is incorrect because in volvulus, blood may be seen in the stool but not typically in the vomitus. Bowel ischemia from volvulus can cause significant abdominal pain. E. Choice E is incorrect because in intussusception we might see the classic "currant jelly" stools. The abdominal exam in children with intussusception often shows the presence of a sausage-like mass due to the telescoped bowel.

You are seeing a 1-month-old infant who is < 3rd percentile for weight. He is breastfed every 2 hours and latches on well. However, he has frequent non-bilious episodes of vomiting that have been increasing over the past week despite his mother taking "reflux precautions." He does not have mucus or blood in his stool. Physical exam reveals a small, olive-sized mass in his abdomen. What is the most likely diagnosis? A. Cleft palate B. Pyloric stenosis C. Cystic fibrosis D. Non-organic failure to thrive E. Munchausen syndrome by proxy

Q15.4 > B has been selected by the expert. A. Choice A is incorrect. Children with structural anomalies of the palate typically present with difficulty feeding. This child appears to be latching on well, but has difficulty retaining the food once ingested. B. Choice B is correct because the history of frequent vomiting, poor weight gain, and the finding of an abdominal mass are consistent with pyloric stenosis. Children with pyloric stenosis often present at 3 weeks of age. C. Choice C is incorrect. Children with cystic fibrosis typically present with failure to thrive secondary to chronic malabsorption, with characteristic loose and malodorous stool. Lab testing reveals elevated sweat chloride. There may be a known family history. D. Choice D is incorrect. Several findings in this patient point toward an organic rather than an inorganic cause of failure to thrive, and organic causes such as pyloric stenosis should be ruled out before an inorganic cause is assumed. E. Choice E is incorrect. Although maltreatment of a child should always be considered, particularly in the youngest children. This child has sufficient findings by history and physical exam to suspect true organic etiology.

A 15-month-old boy presents to the ED in January with a 3-day history of diarrhea. His current weight is 11 kg. He was born at 39 weeks, without any perinatal complications. There is no significant history of travel, sick contacts, or recent changes in diet. The mother notes that he has had only 2 diaper changes over the last day. Physical exam is remarkable for an irritable but consolable infant with tachycardia and normal blood pressure. He is crying without tears and his mucous membranes are dry. His abdominal exam is benign. There is no tenting, and capillary refill is 2 seconds. He is diagnosed with gastroenteritis and started on rehydration therapy. Which of the following statements is true? A. The patient is mildly dehydrated and should be managed with oral rehydration (Pedialyte). B. The patient is moderately dehydrated and should be managed with oral rehydration (Gatorade). C. The patient should be rehydrated with clear liquids and then transitioned to a lactose-free diet until his diarrhea resolves. D. The patient is moderately dehydrated and should be bolused with 220 ccs of D5 ½ normal saline for emergency phase correction, to ensure hemodynamic stability. E. ) The work-up for infectious diarrhea for this patient should include a Wright's stain for fecal WBCs, a stool Rotazyme, and a stool sample for culture and sensitivity.

Q15.5 > E has been selected by the expert. A. The patient is likely moderately dehydrated, given the minimal urine output over the last day, the tachycardia (in the face of normal blood pressure), and the preserved skin turgor and capillary refill. If the patient was mildly dehydrated, oral rehydration therapy would be appropriate management, and in low resource settings, oral rehydration has been used very successfully for moderately dehydrated children as well. B. Gatorade would not be recommended as a rehydrating solution for a 15-month-old child. The sugar concentration is high and this may lead to an osmotic diarrhea. C. Current recommendations from the American Academy of Pediatrics suggest reintroduction of the regular diet, as tolerated, and not transitioning first with a lactose-free diet. D. Moderately dehydrated patients should be bolused with 20 ccs/kg of IV fluid to insure hemodynamic stability and adequate perfusion of vital organs. The preferred fluids would be normal saline or lactated ringers. E. In addition to correcting this patient's hydration status, a work-up for the infectious causes of this patient's diarrhea might include a stool Wright's stain for fecal WBCs (which would suggest a bacterial cause if this is infectious diarrhea), a Rotazyme test (given the high incidence of rotavirus in the winter months), and a stool sample for culture and sensitivity. Additional studies might include stool guaiac (for occult blood) and a check for stool C. diff toxin.

You are seeing a 36-month-old boy for his well-child visit. His parents are anxious about ensuring that his development is appropriate. He passed a hearing screen at birth and, other than a few colds, has been generally healthy. He has never been hospitalized or had any serious illness. He is able to run well, walk up stairs, and walk slowly down stairs. He uses more words than the parents are able to count, but can use them only in short, two or three-word sentences. His speech is understandable. He can draw a circle, but not a cross. Neurologic examination shows normal cranial nerves, normal sensitivity, normal motor reflexes, and no Babinski sign. Which of the following is the most appropriate next step in the management of this patient? A. Perform a brain-stem auditory evoked potential hearing screen B. Perform a screening exam for autism C. Reassure the parents that the boy's development appears normal D. Refer the child to a developmental specialist for comprehensive evaluation E. Refer the child to a specialist for evaluation of his delayed motor development

Q2.1 > C has been selected by the expert. A. A brain-stem auditory evoked potential hearing test (BAER) may be indicated in infants who fail to meet language milestones if they cannot cooperate with other more comprehensive testing. A 36-month-old should be able to cooperate with behavioral audiometry, so a BAER is not indicated. In addition, this child has no evidence of language delay and does not require referral at this point. B. Autism is an increasingly diagnosed cause of developmental delay, but this child is not delayed and no mention is given of any autistic features, such as a lack of symbolic play, repetitive movements, or poor sociability. C. The developmental milestones mentioned in the vignette are within the range of normal for a 36-month-old child. In the absence of any other evidence of significant impairment, there is no indication for referral at this point. D. If there are reasons for concern on developmental screening tests, a referral may be indicated. However, the developmental milestones mentioned in the vignette are within the range of normal for a 36-month-old child. E. This child's motor milestones are not delayed, and no referral is indicated.

Sammy is a healthy male child brought into your office by his mother for a well-child examination. As part of your evaluation you assess his developmental milestones. He is able to run, make a tower of 2 cubes, has 6 words in his vocabulary, and can remove his own garments. What would you estimate Sammy's age to be based upon his developmental milestones? A. 12 months B. 15 months C. 18 months D. 30 months E. 36 months

Q2.2 > C has been selected by the expert. A. At age one year, gross motor skills include pulling to stand, standing alone, and perhaps first steps. Fine motor skills including putting a block in a cup and banging 2 cubes held in hands. At this age a child should be able to imitate vocalizations/sounds and babble. The majority of children this age will know 1 or 2 words in addition to "mama" and "dada." Social-emotional milestones at age one year are waving bye-bye and playing pat-a-cake. Running, building towers of blocks, removing clothing, and a 6-word vocabulary are more advanced skills than a 12-month-old would be expected to have. B. At 15 months of age, a child should be able to stoop and recover and walk well, put a block in a cup, have a vocabulary of a few words, wave bye-bye, and drink from a cup. Running, building towers of blocks, removing clothing, and a 6-word vocabulary are more advanced skills than a 15-month-old would be expected to have. C. At 18 months, a child should be able to walk backward, and 50-90% of children can run at this stage. An 18-month-old should be able to scribble, build a tower of 2 cubes, have 3-6 words in her or his vocabulary, and be able to help in the house and remove garments. D. At 2 ½ years of age, kids can jump up and throw a ball overhand. They can build a tower of 6-8 cubes, point to 6 body parts, name 1 picture, put on clothing, and wash and dry their hands. Sammy is only able to build a tower of 2 cubes, can remove his clothing but does not yet put clothing on, and his vocabulary is limited to 6 words-leading us to believe he is not 2 ½ years old. E. At age 3, children can balance on each foot for 1 second, wiggle their thumbs, name 4 pictures, name 1 color, name a friend, and brush their teeth with help. Sammy's vocabulary is only 6 words, he is not able to name a friend, he is only able to stack 2 cubes, and he has just starting running, but is unable to balance on each foot for 1 second.

Mark is a 5-month-old male who is brought to the urgent care clinic with a three-day history of rhinorrhea and non-productive cough. When he was born he was large for gestational age, and his exam then was notable for macrocephaly, macroglossia, and hypospadias. On physical exam now his vitals signs are stable. He has copious nasal discharge, but his lungs are clear to auscultation. On abdominal exam, you palpate an abdominal mass on the right side just below the subcostal margin. It is 7 cm in diameter and does not cross the midline. The abdomen is soft and non-tender with active bowel sounds. What is the most likely cause of his mass? A. Wilms' tumor B. Teratoma C. Renal cell carcinoma D. Hepatoblastoma

Q2.3 > A has been selected by the expert. A. Wilms' tumor is commonly associated with Beckwith-Wiedemann syndrome, a genetic overgrowth syndrome. Other features that may be seen in children with this syndrome include omphalocele, hemihypertrophy, hypoglycemia, large for gestational age, and other dysmorphic features. B. Teratomas are congenital tumors that are present at birth. These benign tumors that are often identified incidentally, or may become symptomatic due to mass effect of the lesion within the abdominal cavity. The aggressiveness of the tumor depends on the degree of differentiation. C. Renal cell carcinomas are much more common in adulthood. Risk factors include cigarette smoking and obesity. D. While children with Beckwith-Wiedemann syndrome can have hepatoblastoma (in addition to other types of tumors), this is not the most common tumor in this genetic condition. Note that hepatoblastoma may also be associated with familial adenomatous polyposis.

An asymptomatic, healthy 9-month-old female is found to have a palpable RUQ mass on exam. After further imaging and lab studies, the mass is diagnosed as a neuroblastoma that has involvement in the bone marrow as well. The mother is worried about the prognosis. Which of the following is true about the prognosis of neuroblastoma in this child? A. Lymph node involvement is a poor prognostic factor B. Prognosis of neuroblastoma is predictable C. Children who are older than 12 months have a better prognosis than younger children D. Favorable histology does not play a role in prognosis E. Non-amplification of the n-myc gene is a favorable prognostic factor.

Q2.4 > E has been selected by the expert. A. Due to the effectiveness of chemotherapy, neuroblastomas with lymph node involvement are still considered favorable, especially in the setting of other favorable factors, such as young age and differentiating histology. Though distant metastasis is a significant poor prognostic factor, regional lymph nodes do not significantly affect the outcome. B. Neuroblastoma has a broad spectrum of clinical courses. Some tumors may spontaneously regress, some may mature to a benign type, and yet other tumors can be very aggressive with metastases. Age plays a role in the prognosis, as most infants have a good prognosis even with disseminated disease, while infants over 18 months of age do not do as well. C. In infants less than one year of age, neuroblastoma tumors may spontaneously regress. Stage 4S neuroblastoma is a special category that is reserved for infants less than 12 months who have resectable primary tumors and metastases to the liver, skin, and bone marrow. Overall survival is over 85 percent for babies over 6 weeks of age with Stage 4S. D. Favorable histology is a good prognostic factor in neuroblastoma, and is based on the differentiation of the cells involved. E. Non-amplification of the n-myc gene is one of the favorable genetics in neuroblastoma.

A 9-month old baby boy comes to the clinic for a well-child visit. The child is at the 50th percentile for weight, length, and head circumference. He is reaching all developmental milestones appropriately. The mother has no concerns at this visit. The child has previously received the following vaccines: 3 doses of DTaP, 3 doses of Hib, 2 doses of HepB, 3 doses of RotaV, 2 doses of IPV and 3 doses of PCV13, and no influenza vaccines. Which vaccines should the child receive at today's visit? A. Influenza, Hep B, IPV, DTaP B. Influenza, IPV C. Influenza, Hep B, IPV D. Hep B, DTaP, IPV E. Hep B, IPV, and MMR

Q2.5 > C has been selected by the expert. A. Influenza, Hep B, IPV, DTaP is incorrect. All three doses of DTaP have been given. B. Influenza, IPV is incorrect. The patient needs the third Hep B shot. C. Influenza, Hep B, IPV is correct. The patient needs a third Hep B, a third IPV, and a yearly flu shot starting at 6 months of age. D. Hep B, DTaP, IPV is incorrect. All three doses of DTaP have been given and the patient now needs a yearly flu shot starting at 6 months of age. E. Hep B, IPV, and MMR is incorrect. The patient also needs a yearly flu shot starting at 6 months of age and MMR is not given before 12 months of age.

A 10-month-old asymptomatic infant presents with a RUQ mass. Work-up reveals a normocytic anemia, elevated urinary HVA/VMA, and a large heterogeneous mass with scant calcifications on CT. A bone marrow biopsy is performed. Which of the following histologic findings on bone marrow biopsy is most consistent with your suspected diagnosis? A. Sheets of lymphocytes with interspersed macrophages B. Small round blue cells with dense nuclei forming small rosettes C. Hypersegmented neutrophils D. Stacks of RBCs E. Enlarged cells with intranuclear inclusion bodies

Q2.6 > B has been selected by the expert. A. This is incorrect, as sheets of lymphocytes with interspersed macrophages are associated with Burkitt lymphoma. B. This is the correct response. In addition to neuroblastoma, other tumors associated with small blue cells include Ewing's sarcoma and medulloblastoma, both of which tumors are seen in children. C. Hypersegmented neutrophils are characteristic of megaloblastic anemia, a condition associated with a vitamin B12 and/or folate deficiency, not malignancy. D. Stacks of RBCs suggest rouleaux formation, a phenomenon seen in multiple myeloma, a condition not seen in young infants. E. This describes the classic "owl's eyes" seen in CMV and other viral infections.

A 6-month-old infant arrives in the ED with a 12-hour history of poor feeding, emesis, and irritability. On exam, she is ill-appearing with T 39.2 C, P 160 bpm, R 40 bpm, BP 80/50 mmHg. CBC shows WBC 11.2, Hgb 13.5, Plt 250. Urinalysis shows > 100 WBC per hpf, positive leukocyte esterase, and positive nitrites. She has no history of prior urinary tract infection. Chest x-ray is negative. Urine and blood cultures are pending. After bringing her fever down, she was still uninterested in drinking, but her exam improved, and you were confident she did not have meningitis, so an LP was not performed. Which of the following is the best next step in management? A. Oral ampicillin B. Oral ampicillin + gentamicin C. Intravenous ciprofloxacin D. Intravenous ceftriaxone E. Intravenous piperacillin + tazobactam

10.1 > D has been selected by the expert. A. This patient is too sick for oral treatment, so oral ampicillin would be insufficient. Also, there is rising resistance of E. Coli to ampicillin, so secondary coverage with gentamicin or some other aminoglycoside would be needed unless cultures proved the organism was sensitive to ampicillin alone. B. Although parenteral and oral treatment produce similar outcomes in high quality RCTs, this patient is ill and refuses to drink and so requires parenteral antibiotics. IV ampicillin and gentamicin could be an appropriate choice for empiric coverage. C. Ciprofloxacin could be used for complicated UTIs, but it has the potential for adverse reactions in young children so is reserved for patients > 1 year with complications such as resistant organisms or urinary tract anomalies D. This patient's presentation is suggestive of a UTI. Given the ill appearance, vital signs, and white count, Upper tract disease (pyelonephritis) should be strongly considered. A parenteral (IV/IM) third-generation cephalosporin is the best choice of those listed for pyelonephritis, given its excellent gram negative coverage (except for Pseudomonas). E. IV piperacillin + tazobactam has excellent gram negative coverage with added Pseudomonas coverage, but it is highly expensive and Pseudomonas is unlikely to be the cause of a UTI in a child who is not regularly catheterized.

A 3-month-old male presents to the ED with a fever that started the previous day. Mother reports that he was fussy and had decreased oral intake. He had had five fewer diaper changes than usual. He had no vomiting, diarrhea, or respiratory difficulty. On physical exam his temperature is 101.6 F, pulse 110 bpm, RR 24 bpm, and BP 95/67 mmHg. The baby seems irritable and is not consolable by the parent. HEENT exam was significant for dry mucous membranes. Other than his irritability, the rest of the physical exam was unremarkable. CBC showed WBC 3.5, but was otherwise normal. BMP was within normal limits. Urinalysis showed positive leukocyte esterase, positive nitrite, and WBCs > 10/hpf. An LP was performed, and urine and CSF culture results are pending. The patient is placed on IV fluids and is started on cefotaxime. What is the next best step in evaluation? A. Renal bladder ultrasound B. Kidney-ureter-bladder (KUB) x-ray C. Intravenous pyelogram D. VCUG E. Oral ampicillin

10.2 > A has been selected by the expert. A. This infant has a fever without other respiratory symptoms. Meningitis and UTI must be considered in patients with fever. The only way to rule out meningitis is by lumbar puncture. This patient has a low WBC, suspicious for sepsis, and a UA that is highly suggestive of UTI. Empiric therapy should be started to cover common organisms including E.coli, P. mirabilis, and Klebsiella. Cefotaxime is reasonable empiric therapy. Renal ultrasound is recommended for all infants with pyelonephritis to assess for renal structural abnormalities or signs of obstructive uropathy (hydronephrosis). B. KUB is not recommended for UTI. C. Intravenous pyelogram would expose the patient to radiation and would not be recommended to screen for renal abnormalities. D. VCUG screening is recommended only for recurrent UTI or when there is abnormal renal ultrasound. E. The patient is already on parenteral antibiotics, so oral antibiotics would not be necessary. Also, ampicillin would not provide empiric coverage.

A 10-day-old boy is brought to the ED by his mother because of "fever." Mom describes that the baby has been "sleepy" and feeding less vigorously than in the previous two days. She believes his urine output has also decreased. His birth history is notable for prolonged membrane rupture (about 32 hours), and maternal fever at the time of delivery. Prenatal and neonatal ultrasound revealed bilateral hydronephrosis. On exam, the infant is sleepy with a temperature of 38.5 C. A blood sample is sent for CBC, BMP, and culture. Attempts are made to obtain CSF and urine for analysis and culture, but only very small volumes of these fluids are obtained. Volume resuscitation is begun. Chest x-ray is performed with indeterminate results. What is the most appropriate next step? A. Send samples for gram stains and begin parenteral empiric antibiotic treatment B. Send the urine for urinalysis and the CSF for cell count, glucose and protein and begin parenteral antibiotic therapy C. Admit for observation and continue supportive care D. Send samples for culture and begin parenteral antiobiotic treatment E. Attempt to obtain larger samples. Antibiotics should not be started until all needed results are pending.

10.3 > D has been selected by the expert. A. Although sending samples for gram stain may give an indication of whether an infection is present, it will not give the same degree of information as would a culture with sensitivities. B. Urinalysis and CSF profiles may help us make the diagnosis, but if positive in the absence of cultures, will commit us to a prolonged course of broad-spectrum and non-specific therapy. C. Delay of therapy would not be indicated. In an infant younger than one month, fever with any suspicion of sepsis, whatever the source, requires immediate evaluation and initiation of antibiotic treatment. D. Given the presentation of fever in a neonate who presents with sleepiness and poor feeding, samples should be sent for culture and the baby started on empiric antimicrobial therapy. This infant is likely to have a urinary tract infection, and urosepsis is certainly a possibility, especially given his known urinary tract anoamlies. We have no way of ruling out meningitis from this presentation, so antibiotics should be initiated at meningitic dosing. In an infant younger than one month, fever with any suspicion of sepsis, whatever the source, requires immediate evaluation and initiation of antibiotic treatment. Because infants at this age have immature immune systems, they do not localize infections as well as older children. An infection of the urinary tract may lead to bacteremia, which in turn may lead to CNS infection. Only cultures will give us the information required to determine the appropriate type length of antimicrobial therapy. E. Given the consequences of significant bacterial infection in an infant this age, delaying therapy to obtain additional laboratory specimens is not appropriate.

A 6-month-old female is brought into the pediatrician's office for three days of high fever, fussiness, and decreased appetite. The patient has not had any upper respiratory tract symptoms, vomiting, diarrhea, or rash. On physical exam the patient is fussy, has a RR of 28 bpm and a pulse of 160 bpm. She is febrile to 102.8 F (rectal). The patient is alert and fully moving all extremities. Apart from her vital signs, no other significant exam findings are noted. A CBC demonstrates leukocytosis of 17.0 cells x 103 / µL with elevated bands. What diagnosis is most likely? A. Measles B. Bacterial meningitis C. Acute otitis media D. Urinary tract infection E. Roseola

10.4 > D has been selected by the expert. A. Measles typically begins with a "prodrome" period featuring the "3 Cs" (cough, coryza, conjunctivitis)-none of which this patient has-along with high fever, often > 104 F, and general malaise and anorexia. On the 2nd to 4th day a maculopapular erythematous rash appears starting on the face/upper neck and spreading downward. Although infants receive their first vaccination against measles (the MMR) at 1 year of age, infants are generally protected unless they are exposed to older, unimmunized children who have the disease. B. This answer is incorrect in this situation for several reasons. First, this patient is not toxic appearing, nor is her physical exam positive for any findings suggestive of meningitis (such as bulging fontanel or extreme irritability). Unlike older children or adults, classic meningeal signs will often not be present or will be difficult to appreciate on an infant. The patient's CBC is significant for leukocytosis with elevated bands, suggesting a bacterial infection. On exam the patient is alert and responding well to her environment and does not demonstrate lethargy, respiratory distress or signs of ICP. A definitive lumbar puncture may be ordered if there is more suspicion for meningitis or if the patient's status deteriorates in any way, and caretakers should be given a follow-up appointment as well as clear indications of when to seek care. C. This answer is incorrect because while fever and fussiness can be possible signs of AOM in infants, there is no evidence of infection on physical exam such as inflamed, erythematous tympanic membranes, with bulging of the membrane indicating an effusion. D. UTI, the most common bacterial illness in a female infant, is consistent with her high fever, fussiness, and decreased appetite. Her CBC suggests that she has a bacterial infection (leukocytosis and elevated bands). A sample of her urine should be obtained by catheterization and sent for urinalysis and culture. E. Roseola often presents with a high fever, but also often with a viral prodrome. It is a diagnosis of exclusion at this point and should not preclude obtaining a urine sample in this child.

A 6-month-old female with normal birth and developmental history presents with fever for the past two days, fussiness, and decreased appetite. ROS is negative. No abnormalities are noted on the physical examination. A urinalysis from a bag specimen is positive for leukocytes and nitrite, which suggests the presence of a UTI; a culture from this sample is pending. The patient is ill-appearing, dehydrated, and unable to retain oral intake. She is hospitalized, receives a 20 cc/kg NS bolus and is placed on maintenance IV fluids with clinical improvement. What is the best next step for management of this patient? A. Urinary catheterization B. Renal bladder ultrasound C. Begin parenteral antimicrobials D. Midstream clean catch urine collection E. Increase intravenous fluid administration rate to flush the kidneys

10.5 > A has been selected by the expert. A. Urinary catheterization is correct . It is the best method for obtaining a specimen for culture that has not been contaminated by perineal bacteria, and for this ill child, you must determine the cause of the fever with accuracy. B. Renal ultrasound may be indicated if the infant is found to have a urinary tract infection but is not indicated as part of the initial work up. C. With a bag culture pending, you may have a contaminated/confusing culture result and may have to rely solely on clinical criteria to treat this presumed UTI. D. Midstream clean catch specimen is incorrect because it is not ideal in a patient who has not been toilet-trained and cannot void on demand. E. While it is important to provide hydration, this patient has responded well to the initial fluid administration, and there is no evidence that increasing this beyond normal recommendations is helpful.

A 5-year-old girl, previously healthy, presents with an erythematous, vesicular rash on the palms and soles and a high fever for several days. Upon examination, she is also found to have ulcers in her mouth. A few days later, the fever and rash resolve. What is the most likely pathogen? A. Herpes simplex virus 1 (HSV-1) B. HIV C. Enterovirus D. Human herpesvirus 6 (HHV-6) E. Group A strep

11.1 > C has been selected by the expert. A. HSV-1 causes gingivostomatitis and can sometimes be accompanied by fever and malaise, but lesions on the hands and feet would be uncommon. B. HIV infection can increase the risk of oral lesions secondary to infections by HSV-1 or Candida albicans, but again would be unlikely to cause lesions on the hands and feet. Furthermore, at this patient's age (5 years), she is unlikely to be HIV-positive unless it was vertically transmitted from her mother. C. This presentation is consistent with infection by cocksackie A, an enterovirus. Following an incubation period of three to five days, patients have fever, tender vesicles on their hands and feet, and oral ulcers. Sometimes the rash also occurs on the buttocks and the genitals. The infection resolves spontaneously within three days, and is spread from person to person via saliva, fluid from the vesicles, stool, or nasal discharge. D. HHV-6 is the virus that causes roseola, which manifests as fever followed by a macular or maculopapular rash, but this rash begins on the trunk, eventually spreading to the extremities, and does not cause oral lesions. E. Group A strep infection could cause fever and a rash with scarlet fever, but this rash is described as "sandpaperlike" with small papules, not vesicular, and is also not confined to the hands and feet.

A 2-year-old girl presents to the urgent care clinic with a 7-day history of high fever to 38.5 C, a maculopapular rash that began on the palms and soles of her feet, red eyes without discharge, and unilateral cervical adenopathy. What other symptom/sign might you discover on further history and exam? A. Tonsillar exudates B. Headache C. Erythematous and edematous feet D. White spots on buccal mucosa E. Dysuria

11.2 > C has been selected by the expert. A. Tonsillar exudates would be present in strep pharyngitis or tonsillitis. Given the prolonged fever, rash, lymph node involvement, and conjunctivitis, the disease process is more widespread than a simple tonsillitis. You should recognize this constellation of symptoms as Kawasaki disease. B. Headache would be present in Rocky Mountain Spotted Fever. This seems reasonable, given the fever and rash that began on the palms and soles. However, the other findings suggest Kawasaki disease, so this is not the best answer. C. The constellation of symptoms described suggests Kawasaki disease. The other two classic signs not mentioned are erythematous tongue ("strawberry tongue"), and erythema/edema of the extremities, which is the best answer here. D. White spots on the buccal mucosa are also known as Koplik spots, which are pathognomonic for measles. The fever and conjunctivitis could be measles, but there is no cough or coryza. In addition, the rash typically starts at the head and moves downward, rather than starting on the hands and feet. E. Although children with Kawasaki disease can have pyuria, it is not associated with dysuria, a symptom of a UTI, which would be highly unlikely given the other signs.

A 3-year-old boy presents with fever to 103 F for the past week, injected eyes, and a refusal to walk for the past two days. On physical exam, you note conjunctival injection without pus or exudates bilaterally, prominent papillae of his tongue with redness as well as redness of his hands, and feet. He also has a new non-diffuse maculopapular rash on his torso that gets worse with fever. On examination of the swollen extremities, you are unable to elicit any tenderness or effusions in any joints. Which of the following is the most likely diagnosis? A. Rocky Mountain Spotted Fever (RMSF) B. Bone or joint infection C. Kawasaki disease (KD) D. Scarlet fever E. Systemic onset juvenile idiopathic arthritis

11.3 > C has been selected by the expert. A. Rocky Mountain Spotted Fever (RMSF) is a tick-borne disease caused by Rickettsia rickettsii. This tick is commonly found in southeastern parts of the U.S., and patients will often come from or have a history of travel to that region. The disease is characterized by headache, fever, myalgia, and a centrally progressing petechial rash originating on the wrists and ankles. The maculopapular rash, and constellation of other symptoms, as well as lack of any recent travel history, makes this diagnosis less likely in this patient. B. Bone or joint infection should be on the differential given the patient's refusal to walk, as up to 80% of these infections are in the lower extremities. The fever associated with septic arthritis and osteomyelitis typically are not as elevated as the one presented in this case, and the lack of localized symptoms of warmth and tenderness associated with the lower extremity erythema and swelling make this diagnosis somewhat less likely in this patient. C. Kawasaki disease (KD) is one of the most common vasculitides of childhood. For diagnosis, in addition to fever of > 5 days, patient must meet four of the following criteria: rash, conjunctivitis, unilateral cervical lymphadenopathy, changes in oral mucosa, or extremity changes (redness/swelling). Our patient does not have lymphadenopathy, but often this is the least common finding in KD. If children have fever with fewer than four of the five clinical findings, they can have "incomplete KD" if they meet certain laboratory criteria. D. Scarlet fever is caused by erythrogenic toxin produced by Streptococcus pyogenes. Symptoms can include sore throat, fever, "strawberry tongue" and a blanching, erythematous rash with desquamation of the affected areas about six to seven days later as the rash begins to disappear. While our patient does have a "strawberry tongue," fever, and rash, the description of the rash and other physical exam findings are more consistent with KD than with scarlet fever. E. Systemic onset juvenile idiopathic arthritis, also known as Still's disease, is a subset of JIA describing patients with intermittent rash, fever and arthritis. While our patient does present with rash and fever, as well as refusal to walk (potentially a sign of arthritis), systemic onset JIA tends to present with a history of spiking fevers and "salmon" rash occurring when the child is febrile, and disappearing as the fever fades. This is inconsistent with the description of the findings seen in our patient, who does not demonstrate tenderness or effusion in any joint.

A 5-year-old boy comes to the clinic with a chief complaint of four days of progressively worsening fever and that has been minimally responsive to acetaminophen. The patient complains of sore throat and decreased appetite. His sister had a positive rapid strep test and is now being treated with amoxicillin. Your concern is for Group A strep. What is the next best step in management? A. Start antibiotic treatment B. Send blood cultures C. Advise parents to give patient acetaminophen with return precautions D. Rapid strep test with back-up culture if negative E. Chest x-ray

11.4 > D has been selected by the expert. A. Choice A is incorrect. Although you may empirically treat this child for infection with Group A strep, a test to diagnose infection should be done prior to initiation of antibiotics. B. Choice B is incorrect. There is no indication for a blood culture at this time. C. Choice C is incorrect. As the patient has a history of being exposed to a sick contact with Group A strep, being sent home with acetaminophen is also not sufficient because the patient has already been treated at home with acetaminophen with no improvement. D. Choice D would provide confirmation of your clinical suspicion and allow for correct diagnosis prior to empiric antibiotic treatment. E. Choice E is incorrect because there is no indication of respiratory symptoms.

A 3-year old girl comes to the clinic with a chief complaint of fever (104F) for over a week. Her mom reports that she has been fussy and inconsolable since she became febrile. She has a red tongue, with large papillae, conjunctivitis, a palmar rash, unilateral cervical adenopathy, as well as swollen feet. Given the most likely diagnosis, what is the most important follow-up for this girl over the next few weeks? A. Ophthalmology follow-up to determine extent of eye damage and determine need for corticosteroids B. Physical therapy follow-up to help prevent long-term joint deformities and ensure long-term functionality C. Cardiology follow-up to rule out presence of rheumatic fever D. Echocardiogram to look for coronary artery aneurysm E. Neurology follow-up to evaluate partial paralysis of lower extremities

11.5 > D has been selected by the expert. A. Choice A is incorrect because the patient likely has Kawasaki disease. Early referral to an ophthalmologist is important for patients diagnosed with other conditions, including Stevens Johnson Syndrome (SJS) to determine the degree of eye involvement and if treatment with topical steroids is needed. SJS is a mucocutaneous disorder defined by fever, severe stomatitis (inflammation of the mucous lining of any of the structures in the mouth), conjunctivitis, and a blistering rash. It is typically caused precipitated by medications or infections. Early referral to an ophthalmologist would also be important for patients diagnosed with juvenile idiopathic arthritis, because they can suffer from uveitis, which if left untreated, can lead to long-term problems such as cataracts, glaucoma, or blindness. B. Choice B is incorrect because the patient in this case has Kawasaki disease. Early referral to physical therapy is important for patients diagnosed with systemic juvenile idiopathic arthritis, which is characterized by prolonged or spiking fever, rash, and arthritis. Early treatment and physical therapy can help prevent joint deformities and improve long-term functionality. C. Choice C is incorrect because the patient in this case has Kawasaki disease. Patients who have scarlet fever could develop rheumatic fever or post-streptococcal glomerulonephritis, among other problems. Scarlet fever is characterized by a "erythematous, blanching, sandpaper-like rash" with very fine papules secondary to infection with Group A streptococcus. It may start in the groin, axilla, or neck, before spreading rapidly over the trunk and extremities. Fever can be high, but generally resolves within five days. D. Choice D is correct because children with Kawasaki disease are at high risk for coronary artery aneurysm formation and should receive an echocardiogram within four weeks of the onset of their illness. Use of IVIG for the treatment of Kawasaki disease has decreased the risk of coronary artery aneurysms significantly. Kawasaki disease is diagnosed when there is a fever plus four of the following: changes in oral mucosa (e.g., strawberry tongue), extremity swelling or redness, unilateral cervical adenopathy, conjunctivitis, and rash. Infectious and rheumatologic causes must be excluded in order to make the diagnosis of Kawasaki disease. E. Choice E is incorrect because the patient in this case has Kawasaki disease. Neurologic issues such as paralysis of a lower limb can occur in severe cases of Rocky Mountain Spotted Fever, which is characterized by fever, myalgias, headache, and petechial rash classically starting on wrists and ankles and progressing centrally.

A 12-year-old boy presents to the ED with complaints of anorexia, weight loss, and persistent cough, with nocturnal coughing fits that have been waking him from sleep for the past three weeks. He denies fever, chills, myalgia, sore throat, or rhinorrhea. The patient presented to his primary care physician one week prior with the same complaint, and was treated with amoxicillin and bronchodilator therapy. His chest x-ray was negative for infiltrates at that visit. The patient's symptoms did not improve with this regimen. The cough became more frequent, sometimes causing emesis. Which of the following is the most likely diagnosis? A. Reactive airway disease B. Infection with Bordetella pertussis in the catarrhal stage C. Infection with Bordetella pertussis in the paroxysmal stage D. Atypical pneumonia due to Mycoplasma pneumoniae E. Laryngotracheobronchitis

12.1 > C has been selected by the expert. A. Reactive airway disease would most likely have improved with bronchodilator therapy. In addition, you would not expect to see anorexia or weight loss with reactive airway disease. B. The catarrhal stage of pertussis lasts one to two weeks and is often indistinguishable from URI. In this patient the presenting symptoms have been an ongoing problem for more than three weeks. C. The paroxysmal stage of pertussis lasts four to six weeks and is characterized by repetitive, forceful coughing episodes, followed by massive inspiratory effort. This massive inspiratory effort is what results in the characteristic "whoop"-sounding cough. This is consistent with the patient's presentation and duration of illness. The forceful coughing fits in pertussis can even lead to conjunctival hemorrhages and pneumothoraces from the increased intrathoracic and intracranial pressures from Valsalva. The antimicrobial agents of choice for treatment of pertussis are azithromycin, clarithromycin, and erythromycin. Antibiotics given in the paroxysmal phase will reduce communicability but will not alter the clinical course. D. Mycoplasma pneumonia would be expected to be associated with fevers and findings on chest x-ray and lung exam. E. Croup (laryngotracheobronchitis) presents with difficulty breathing and a "seal bark" cough and usually lasts a week or less. Croup also is usually associated with fever.

A 12-month-old previously healthy girl presents with cough and mild subcostal retractions. She is afebrile, and physical exam reveals asymmetric wheezing. Chest x-ray demonstrates unilateral air trapping. What is the most likely diagnosis? A. Croup B. Pneumonia C. Acute bronchiolitis D. Foreign body aspiration E. Asthma

12.2 > D has been selected by the expert. A. Croup involves subglottic inflammation, typically presenting with inspiratory stridor and a "barky cough" (i.e., like a seal). This patient is not noted to have either, and also presents with asymmetric wheezing and air trapping that would not be expected in an individual with croup. B. While this patient presents with cough and increased work of breathing, she is afebrile, and auscultation of the chest does not reveal crackles or decreased breath sounds/area of consolidation, which would be consistent with pneumonia. Additionally, chest x-ray findings are not consistent with a lobar or more diffuse pneumonia. C. Acute bronchiolitis is a good thought, especially as this is the most common cause of wheezing in infant; however, if this were the diagnosis, the patient would most likely be febrile and chest x-ray would demonstrate scattered atelectasis and/or diffuse opacities from bronchial obstruction. D. Features of foreign body aspiration include unexplained wheezing and asymmetric breath sounds, as well as air trapping in one lung indicating unilateral airway obstruction. The right main bronchus is the more commonly obstructed due to anatomy (it is wider and more vertical than the left). The most commonly aspirated foods are hot dogs, nuts, hard candy, grapes, and popcorn. E. While the finding of wheezing is consistent with asthma, this patient has wheezing only on one side. Along those lines, chest x-ray in an asthmatic patient would demonstrate global air trapping with hyperinflated lungs, rather than unilateral findings.

A 10-month-old infant is brought to the Peds ED by her parents, who say she has been coughing persistently for the last three hours. The parents were watching a movie at home when they first noticed their daughter coughing. Patient is a vaccinated, well-nourished infant in moderate distress with retractions, nasal flaring, and grunting. On auscultation, you immediately notice diminished breath sounds in the right lung with normal breath sounds on the left. What other associated physical exam finding do you expect to hear? A. Stridor B. Asymmetric breath sounds and wheezing C. Rhonchi D. Crackles E. Bronchial breath sounds

12.3 > B has been selected by the expert. A. Stridor is due to airway narrowing above the thoracic inlet and can be seen in URI such as epiglottitis. You might expect stridor in laryngomalacia, but this would not have suddenly appeared at 10 months of age. Also, epiglottitis is more rare now in a vaccinated child, and you wouldn't expect to hear unilaterally diminished breath sounds. B. This infant is in respiratory distress from foreign body aspiration, consistent with the history of acute onset of distress and asymmetric breath sounds. Common foreign bodies include peanuts, popcorn, grapes, hard candy and hot dogs. Respiratory distress from foreign body aspiration is usually accompanied by asymmetric breath sounds and wheezes on auscultation. C. Rhonchi are coarse, low-pitched, rattling sounds due to secretions and airway narrowing and are typically heard in the setting of bronchitis or pneumonia. D. Crackles are due to fluid in alveoli or opening and closing of stiff alveoli, not consistent with a foreign body aspiration. You would expect this in either pneumonia or CHF from pulmonary edema, both of which are unlikely in this patient given the abrupt onset and lack of history of cardiac problems. E. Bronchial breath sounds are hollow-sounding and caused by air moving through areas of consolidated lung, such as in the setting of pneumonia.

Susie is a 3-year-old girl brought into the clinic by her mother because she has a gradually worsening cough and she has been having trouble breathing. Her mother says Susie sounds like she is barking when she coughs. Susie is up to date with her vaccinations. Susie's mom always watches her when she's playing. On physical exam, you note that Susie has inspiratory stridor. She does not have wheezing, there are no retractions, and she has symmetrical breath sounds. No pseudomembranes are appreciated on physical exam. What is Susie's most likely diagnosis? A. Epiglottis B. Croup (laryngotracheobronchitis) C. Pertussis D. Pneumonia E. Foreign body aspiration

12.4 > B has been selected by the expert. A. Epiglottitis is a life-threatening emergency caused by an infection with H. influenzae type B. It is less common now with the advent of Hib vaccine, but in rare cases can occur due to staphylococcal or streptococcal infections. It most often occurs in children ages 2 to 5 years. Children with epiglottitis present with fever, stridor, drooling, dysphonia, dysphagia, and respiratory distress. They frequently appear toxic and sit in the "sniffing position" (sitting, leaning forward, neck hyperextended, chin protruding). A "thumb sign" (thickened epiglottis and aryepiglottic folds) appears on films. Susie is not exhibiting any of these characteristic symptoms and she is up to date with vaccines, making epiglottitis a less likely diagnosis for her cough. B. Croup or laryngotracheobronchitis is due to a viral infection (Parainfluenza type 1). It is most common in the winter, and often occurs in children age 2 to 5 years. Croup can lead to non-specific URI symptoms with some degree of airway obstruction. A barky or seal-like cough and inspiratory stridor (which should be differentiated from expiratory wheezes) is common in croup. C. Pertussis occurs in three phases: The catarrhal stage lasts one to two weeks when children present with URI symptoms. A paroxysmal stage follows and lasts four to six weeks. During this phase, children have repetitive forceful coughing with massive inspiratory effort ("whoops"). Finally, during the convalescent stage, children present with continued cough that may last up to three months. The acellular pertussis vaccine protects against pertussis, and, as Suzie is up to date with her immunizations, this diagnosis is less likely. Her barking cough is more suggestive of croup than the "whooping" cough of pertussis. D. While retractions and cough are present with pneumonia, asymmetric breath sounds and tachypnea would be more specific for a diagnosis of pneumonia. E. Symmetrical breath sounds and the gradual onset of Susie's cough make foreign body aspiration less likely. One might also expect focal wheezing and tachypnea with foreign body aspiration.

Joe, a previously healthy 11-month-old infant with 5-day history of a "cold," is brought to the ED by mom for one day of acute worsening cough and intermittent wheezing. Per mom, the cough was initially dry but has become more "phlegmy," making it difficult for Joe to breathe, particularly when he is feeding or more active. His immunizations are up to date, and he has no known allergies. His family history is significant for a 6-year old sister who was diagnosed with asthma four years ago. On exam, Joe is afebrile, mildly tachypneic with normal O2 saturation. He has prominent nasal flaring and mild subcostal retractions. He has clear rhinorrhea but no evidence of oropharyngeal erythema. Lung exam reveals decreased breath sounds and wheezes on the right. What is the most likely diagnosis? A. RSV bronchiolitis B. Epiglottitis C. Viral URI D. Asthma E. Foreign body aspiration

12.5 > E has been selected by the expert. A. Bronchiolitis is a lower respiratory tract infection most commonly caused by RSV, which is characterized by bronchiolar obstruction secondary to mucus plugging, cellular debris, and edema. Patients generally present with fever and URI symptoms which progress to a worsening cough, wheezing and shortness of breath. Although this patient does have wheezing, the unilateral wheezing with decreased breath sounds is not consistent with bronchiolitis. B. Epiglottitis was commonly caused by Haemophilus influenzae type B, but can also be caused by Staph and Strep species. Patients may present with fever, dysphagia, drooling, stridor and significant respiratory distress. Patients are generally seen sitting, leaning forward with the neck hyperextended. Epiglottitis has become less common due to immunization with Hib. This diagnosis is less likely in our patient, since his immunizations are up to date, he is afebrile and not in severe respiratory distress. C. Our patient probably developed a viral URI five days ago. An upper respiratory tract infection in children can manifest as fever, rhinorrhea, cough, sore throat and myalgias, and may be accompanied by wheezing. However, our patient's ausculatation findings cannot be explained solely by a viral URI. D. Asthma is caused by inflammation of airway mucosa, mucus hypersecretion, mucosal edema and reversible bronchoconstriction. It generally presents as cough, wheezing, tachypnea and dyspnea worsened by cold air, exercise, allergies and URIs. The mainstay of treatment involves bronchodilators (beta-2 agonists) and inhaled steroids. Asthma is a possible diagnosis in Joe given the family history of asthma; however, it is less likely since he was previously healthy with no history of recurrent cough or wheezing. Furthermore, asthma does not generally present with focal wheezing as heard on Joe's lung exam. E. Given Joe's age, foreign body aspiration should always be included in the differential diagnosis for acute onset wheezing. The lung findings of asymmetric breath sounds and wheezing support this diagnosis. Foreign body in the airway can be confirmed by bilateral decubitus or inspiratory/expiratory chest films, characterized by decreased deflation on the affected side. If complete obstruction, x-ray will generally reveal atelectasis (whiting out) and signs of volume loss (mediastinal shift towards affected side to compensate for loss of volume).

A 4-year-old boy who recently emigrated from eastern Europe presents with his mother to your general pediatrics clinic. His mother reports that he has a chronic nonproductive cough during the day and night, mild wheezing for one month and failure to gain weight (his weight has dropped from the 50th to the 10th percentile for his age). His mother denies any high fevers, rhinorrhea, or night sweats. Which of the following are the next best diagnostic tests? A. Chest x-ray and tuberculin skin test B. CT of nasal sinuses C. Spirometry, before and after bronchodilator therapy D. Chest x-ray and methacholine challenge E. None needed, patient likely has habitual cough

13.1 > A has been selected by the expert. A. CXR and tuberculin skin test (TST) is the best choice. Signs and symptoms of primary pulmonary tuberculosis are few to none. Toddlers may present with nonproductive cough, mild dyspnea, wheezing, and/or failure to thrive (defined as weight < 5th percentile or drop in two percentile curves for weight). In children, TB can present without systemic complaints (fever, night sweats, and anorexia), severe cough, and sputum production. Regarding diagnostic tests, the TST is a practical tool for diagnosing TB infections. All children with chronic cough (more than three weeks) should be evaluated with a chest x-ray, as other pathology-such as lung abscess or malignancy-can also be detected on CXR. B. Sinusitis is often preceded by a URI, with nasal congestion as a prominent feature, leading to nocturnal cough due to post-nasal drip. These symptoms are not seen in our patient. Furthermore, a diagnosis of sinusitis is made clinically, with CT scan obtained only in complicated cases or cases resistant to treatment. Complications include cavernous sinus thrombosis, meningitis, and epidural abscess. C. Spirometry (pulmonary function testing) before and after bronchodilator therapy is the most specific means of determining whether or not a child has reactive airways. Asthma is a very common diagnosis in pediatrics, and may present with cough that is worse at night and exacerbated by exercise and cold air. Patients with cough-variant asthma present with only cough, typically nonproductive. However, given this patient's failure to thrive, a more serious diagnosis such as TB must be considered. Also, a chest x-ray is needed in all children with chronic cough (more than three weeks). D. Although a chest x-ray is appropriate in all children with chronic cough, a methacholine challenge (for asthma) would be inappropriate in this scenario. Although asthma is a common diagnosis, given the patient's failure to thrive, a more serious diagnosis must be considered. Further, a methacholine challenge is reserved for cases in which asthma is suspected and spirometry is normal or near normal, and should be performed by trained individuals. E. Habitual cough is caused by habitual perpetuation of a cough that begins with a viral URI. Continued coughing further irritates the airway, leading to stronger stimulation to cough. The cough is typically very loud, short, dry, brassy, and spasmodic. This cough is unchanged by exercise or cold air, and classically resolves during sleep. Although the patient in this case has a dry cough, his failure to thrive points to a more serious diagnosis (e.g., TB). All children with chronic cough (persisting longer than three weeks) need a CXR.

An 11-year old boy presents to clinic with wheezing. Mom states that in the past he has used inhaled albuterol and it has helped with wheezing and shortness of breath. On further history you find out that the patient experiences shortness of breath three times a week and is awakened at night by these symptoms once a week. What is the most appropriate outpatient therapy? A. Only rescue inhaler PRN B. Low dose inhaled corticosteroids C. Medium dose inhaled corticosteroids and course of oral corticosteroids D. Medium dose inhaled corticosteroids, LABA, and course of oral corticosteroids E. Course of oral corticosteroids

13.2 > B has been selected by the expert. A. Rescue inhaler (a short-acting beta agonist) i PRN is incorrect because this treatment is indicated in patients with intermittent asthma and have symptoms fewer than two days a week or two nights a month B. Low dose inhaled corticosteroid is correct because this patient has mild persistent asthma. His symptoms occur 3-6 days/week and 3-4 nights/month. C. Medium dose inhaled corticosteroids with a course of oral corticosteroids is incorrect, because it would be indicated in a patient with moderate persistent asthma when symptoms occur daily and more than one night per week. D. Medium dose inhaled corticosteroids, LABA, and oral corticosteroids is incorrect because this patient does not have severe persistent asthma. E. A course of oral corticosteroids alone is incorrect. Asthma needs to be managed long term to prevent exacerbations. An inhaled corticosteroid is indicated.

A 4-year-old patient presents with several months of cough. Mom also reports a history of red skin patches, which are pruritic, and allergies to peanuts, eggs, and mangoes. Which of the following would be characteristic of the cough that this patient would present with? A. Does not awaken patient from sleep B. Paroxysmal C. Barking cough D. Worse at night E. Associated with crackles on exam

13.3 > D has been selected by the expert. A. This choice is incorrect. This patient has asthma, which commonly presents with symptoms awakening the patient from sleep. Habitual cough disappears at night. B. This choice is incorrect. Paroxysmal coughs are associated with bacterial infections such as pertussis, Chlamydia, or mycoplasma. Foreign bodies can also produce sudden onset of cough. History could help to determine if the latter is a cause. C. This choice is incorrect. Barking coughs are associated with croup or other forms of subglottic disease. Foreign bodies can also produce this type of cough. D. This choice is correct. Asthma frequently presents with nighttime exacerbations. The cough often presents with wheezing and is usually a dry cough. E. This choice is incorrect. Diseases associated with crackles usually have intrinsic pulmonary involvement. Crackles can be fine or coarse and usually represent alveolar or small airway conditions.

A 9-year-old boy presents to your clinic with discoloration under his eyes, persistent cough, and skin rashes. He is found to have wheezing on physical exam and increased lung volume bilaterally on chest x-ray. He has struggled with these complaints over the past three years but recently his symptoms have gotten worse, affecting him every other day. He is afebrile. He is found to have wheezing on physical exam and increased lung volume bilaterally on chest x-ray. What would be the most appropriate treatment for him? A. Oral antibiotics B. Short-acting beta agonist PRN C. Short-acting beta agonist PRN with low-dose inhaled corticosteroid D. Short-acting beta agonist PRN with medium-dose inhaled corticosteroid E. Long-acting beta agonist

13.4 > C has been selected by the expert. A. The patient's presentation is more consistent with asthma than an infection. The patient has had these complaints for the last few years. His skin rashes and lower eyelid darkening are consistent with allergic processes (atopy), which are associated with asthma. Furthermore, increased lung volumes bilaterally and persistent cough without fever also suggest asthma, thus antibiotics would not be appropriate. B. Cough and wheezing that occur intermittently (< 2 days/week) are consistent with intermittent asthma, which is treated with short-acting beta agonist PRN. C. Persistent cough and wheezing that affect the patient every other day (3-4 days with symptoms/week) are consistent with mild persistent asthma, which is appropriately treated with short-acting beta agonist PRN and low dose inhaled corticosteroid. The swelling under the eyes (allergic "shiners") and skin rash are other signs of atopy, as mentioned above. D. Short-acting beta agonist PRN with medium dose inhaled corticosteroid is the preferred treatment for moderate or severe persistent asthma, which corresponds to daily symptoms or symptoms throughout the day. E. The use of a long-acting beta agonist is reserved for severe persistent asthma, which corresponds to symptoms throughout the day.

A 10-year-old boy comes to the clinic with a chief complaint of progressive cough for two weeks that began gradually. His cough is described as productive and wet with whitish sputum. His mother denies throat pain, vomiting, and diarrhea in his review of systems. His mother reports that he has been febrile up to 101.5°F daily. She thinks he is fatigued and has not eaten well in the past week. On exam, there is air passage throughout all lung fields, with crackles in the lower right lung field, but no other abnormal sounds. What would you likely find in your workup? A. Response to inhaled beta-agonist B. Hyperinflation in one lung field C. Alevolar consolidation in the RLL D. Positive PCR for pertussis E. Fluffy bilateral infiltrates and a large heart on chest x-ray

13.5 > C has been selected by the expert. A. Response to an inhaled beta agonist is a good test for RAD or asthma. Asthma is diagnosed clinically, usually in a school-aged child, with a history of recurrent wheezing. Associated findings might atopic stigmata, such as allergic rhinitis, food allergy, and atopic dermatitis. Without details in the history, asthma is a less likely diagnosis in this setting, and in the absence of wheezing, asthma is even less likely. B. This would be consistent with a foreign body aspiration which could produce cough and fever (if bacterial superinfection occurs). This diagnosis is usually considered in younger children. Constitutional symptoms (fatigue, decreased eating) make pneumonia a better diagnosis in this clinical setting. C. Pneumonia is the most likely cause for his symptoms and a chest x-ray would be a great confirmation of your suspected diagnosis. Eliciting a complete history might reveal history of an upper respiratory infection. Localization of crackles (discontinuous inspiratory sounds) to one lobe makes pneumonia more likely. D. Pertussis can produce a lengthy cough illness, but is not associated with fever or lung findings. E. Pulmonary edema due to CHF is a symmetrical process and less likely to present with a unilateral lung finding. Pulmonary edema should be suspected with crackles, but this clinical setting leaves pulmonary edema low on our differential.

Rosy is an 18-month-old previously healthy baby girl who presents to clinic with congestion for three days. Today, her vitals are: T 101.2°F, BP 100/60 mmHg, P 80 bpm, RR 28 bpm. On physical exam, Rosy has clear mucus coming from both nostrils. Both turbinates show erythema. Her oropharynx is erythematous. No crackles or wheezing are heard. Mom reports that acetaminophen aids in bringing down the fever temporarily; however, the fever returns in a few hours. Mom is concerned for possible pneumonia since she was recently was given antibiotics for bronchitis. Her immunizations are up to date. Which of the following is most likely responsible for Rosy's symptoms? A. Strep pnuemoniae B. Group A Strep C. Rhinovirus D. Hemophilus Influenzae type B E. Pertussis

14.1 > C has been selected by the expert. A. Rosy's symptoms do not fit a diagnosis of pneumonia, as she does not have significant fever, increased respiratory rate, crackles, or rales. B. Group A Strep is rarely a cause of pharyngitis in young children and notably does not cause the feared sequelae (rheumatic fever) in those < 3 years. C. Rhinovirus causes the common cold and is the most reasonable diagnosis. Rhinovirus is a very common cause of congestion and other cold-like symptoms. Rosy presents with slightly elevated temperature, slight tachypnea, and inflamed turbinates and oral mucosa. Her symptoms all correlate with the common cold. D. Hemophilus Influenzae type B causes pneumonia and epiglottitis. Rosy does not have the typical symptoms of epiglottitis that include difficulty breathing, high fever and drooling. With the advent of vaccinations, Hib infections have decreased significantly. E. The catarrhal phase of pertussis can be indistinguishable from the common cold, but quickly develops into the paroxysmal phase. The paroxysmal phase is characterized by coughing fits and post-tussive emesis, which again Rosy does not have. It typically does not have associated fever.

A 14-month-old girl with no significant past medical history presents to clinic with fever to 39.2 C and irritability. According to mom, the patient was initially sick one week ago with a runny nose and cough, but these symptoms had resolved. She started pulling at her ear and becoming increasingly irritable last night, with her fever spiking around 2:00 a.m. this morning. Patient is up to date on immunizations, and has had several prior ear infections. She was most recently treated last month with amoxicillin. When you examine her ears, you observe a red, bulging tympanic membrane with limited mobility in her left ear. The exam of the right ear is normal. You are confident in your diagnosis of acute otitis media. What is your treatment plan? A. Observation B. Anthistamines and decongestants C. High-dose amoxicillin D. Amoxicillin/clavulanate (with high-dose amoxicillin component) E. Tympanocentesis

14.2 > D has been selected by the expert. A. This choice is incorrect due to the patient's presentation with a high fever above 39 C. Fifty to 80% of acute otitis media cases will resolve spontaneously without antibiotics; however, the decision to defer treatment with the "observation option" is based on the child's age and illness severity. This option is limited to healthy children between the ages of 6 months to 2 years with non-severe symptoms. Our patient is presenting with severe symptoms and a high-grade fever. B. This choice is incorrect because the FDA has discouraged the use of over-the-counter cough and cold products in children younger than 2 years due to the increased risk/benefit ratio. Also, these medications would be most useful for the upper respiratory symptoms preceding the ear infection. C. High-dose amoxicillin is the most common first-line treatment for acute otitis media due to its general effectiveness against susceptible and partially resistant S. pneumo, in addition to being low cost and having a high safety profile. However, this antibiotic was recently administered, raising concerns for a resistant organism. D. This choice is correct because of the severe symptoms our patient is exhibiting with a high temperature greater than 39 C. Amoxicillin/clavulanate is the treatment of choice for patients with moderate to severe otalgia or high fever, and is used for additional beta-lactamase coverage for Haemophilus influenzae and Moraxella catarrhalis, and when failure with amoxicillin is suspected. E. This choice is incorrect because the patient has not had recurrent episodes of otitis media and has not started antibiotic therapy. Tympanocentesis is recommended as a diagnostic measure to confirm a bacterial etiology after a patient has failed repeated courses of antibiotics or if an unusually resistant organism is suspected.

An 18-month-old girl is brought to her pediatrician by her mother who notes that she has been has been fussy for the past three days and has been pulling on her ears. The child is up to date with her hepatitis B, rotavirus, DTaP, H. influenza type B, pneumococcus, and polio vaccines. Her temperature is 102.2 F. Otoscopic exam of her left ear shows a yellow, opaque, and bulging tympanic membrane. Which of the following organisms is the most likely cause of the child's condition? A. Streptococcus pyogenes B. Candida albicans C. Haemophilus influenzae D. Rhinovirus E. Moraxella catarrhalis

14.3 > C has been selected by the expert. A. The child is suffering from acute otitis media (AOM). S. pyogenes is a rare cause of this condition (< 5% of cases). One should not confuse this species of strep with S. pneumoniae, which is a common cause of AOM (25-50% of cases). B. Candida albicans is not a frequent cause of AOM. An infection involving this organism in the middle ear should immediately raise suspicion for an immunocompromised state. C. H. influenzae is a frequent cause of AOM (15-52% of cases). Although the child has been vaccinated against H. influenzae type B, this does not cover the unencapsulated strains of H. influenzae that cause AOM. D. Rhinovirus is a potential cause of AOM, but bacterial AOM occurs with a much higher frequency. E. M. catarrhalis is another common cause of AOM. However, this organism is responsible for only between 3% and 20% of cases, making it a less frequent cause than non-typeable H. influenzae.

An 18-month-old presents with yellow and poorly mobile tympanic membranes. Four months prior he presented then with several days of nasal congestion, cough, decreased eating and ear tugging. His exam then revealed a red, nonmobile tympanic membrane and he was treated with amoxicillin. Based on the history and physical exam, what is the most likely diagnosis now? A. Mastoiditis B. Acute otitis media C. Otitis media with effusion D. Otitis externa E. Viral encephalitis

14.4 > C has been selected by the expert. A. Although mastoiditis presents with tympanic membrane changes, the signs of bilateral yellow and poorly mobile tympanic membranes are more suggestive of otitis media with effusion. B. The course of development of this infection over four months makes this diagnosis unlikely. C. The earlier diagnosis of acute otitis media together with current findings of bilateral yellow and poorly mobile tympanic membranes on physical exam make this the most likely diagnosis. D. Although otitis externa presents with otalgia, the sings of bilateral yellow and poorly mobile tympanic membranes are more suggestive of otitis media with effusion. E. Viral encephalitis typically causes neurologic dysfunction with signs that include fever, headache, nausea, vomiting, photophobia, altered level of consciousness, seizures, and possibly focal neurologic signs. This child's presentation is much more consistent with otitis media with effusion.

An 8-year-old girl comes to the clinic with a chief complaint of a "cold" for the past two weeks. On further questioning, she developed a fever of 38.7°C, purulent nasal secretions, malodorous breath, and a nocturnal cough three days ago. Examination of the nose reveals pus bilaterally in the middle meatus, and tenderness over the mid-face. Which of the following is the most likely diagnosis? A. Allergies B. Maxillary sinusitis C. Asthma D. Frontal sinusitis E. Middle ear infection

14.5 > B has been selected by the expert. A. Allergies are a very common diagnosis in pediatrics, and nocturnal cough is a frequently associated symptom. However, allergies usually cause clear, thin nasal secretions and are usually not associated with fever and malodorous breath. Other physical exam findings associated with allergies may include allergic shiners, cobblestoning of the posterior pharynx, and edematous turbinates. B. The maxillary and ethmoid sinuses are large enough to harbor infection in infancy. The sphenoid sinuses do not become large enough until the third to fifth year of life, and the frontal sinuses are rarely large enough until the sixth to tenth year of life. Sinusitis is characterized by the findings in the question stem, and is often preceded by a URI. Pus draining from the middle meatus is suggestive of either maxillary, frontal, or anterior ethmoid sinusitis. C. Asthma, another common diagnosis in pediatrics, is often triggered by a URI, and is frequently associated with a nocturnal cough. Fever, purulent nasal secretions, and malodorous breath are not caused by asthma. End-expiratory wheezing, atopic diathesis, and a positive family history would support a diagnosis of asthma. D. Distinguishing between the different locations of sinusitis can be clinically challenging, especially in a 9-year-old patient who likely has mature sinuses. However, frontal sinusitis is characterized by pain over the frontal bone and perhaps facial swelling in an older child or adolescent. It is less likely to cause the symptoms described here. E. Acute otitis media is a common diagnosis in pediatrics. It can follow a URI or occur simultaneously with sinusitis. However, this patient does not complain of ear pain.

Luanne is a 15-year-old female with three hours of abdominal pain and two episodes of non-bilious, non-bloody vomiting. She rates her pain at 8/10 and describes it as constant and located mainly in the middle of her belly, but somewhat present throughout her abdomen. It is worse with coughing and moving. She has never had this pain before, and has had no appetite since the pain started. She is sexually active with her boyfriend of three months, always uses condoms, and has not been tested for STIs. Her last menstrual period was two weeks ago. Vitals: 37.9 C, HR 100 bpm, BP 120/85 mm Hg, RR 14 bpm. On exam, she exhibits involuntary guarding, mild rebound tenderness, and tenderness to palpation between her right anterior superior iliac spin and umbilicus. On pelvic exam, she reports tenderness when attempting to palpate her right adnexa, but no masses are appreciated and there is no cervical motion tenderness. Her WBC and CRP are within normal limits. Based on the information above, what is the most likely diagnosis? A. Ovarian torsion B. Pelvic inflammatory disease C. Ectopic pregnancy D. Appendicitis

16.1 > D has been selected by the expert. A. Ovarian torsion is more common in the post-menopausal population, though it can present in any age group. It is described as intermittent stabbing pain in the lower abdomen or pelvis. Torsion is often secondary to an ovarian mass, such as a neoplasm or corpus luteal cyst, which may occasionally be appreciated on exam. Nausea and vomiting are very common findings as well. Ultrasound is essential to initial workup. Given Luanne's pain localized around her belly button, her tenderness at McBurney's point, and lack of palpable masses on pelvic exam, ovarian torsion is a less likely diagnosis. B. Pelvic inflammatory disease is definitely a possibility given Luanne's sexual history, lack of STI screening in the past, and adnexal pain. However, this pain is often post-coital and also first occurs during or immediately following menstruation. Another key finding is mucopurulent discharge and cervical motion tenderness, both of which are absent in Luanne. RUQ pain and a fever (present in 50% of patients with PID) are other signs that are not reported in this case. C. Patients with an ectopic pregnancy typically present with painless vaginal bleeding six to eight weeks after their last menstrual period. The pain is described as crampy pelvic pain, and it is often associated with nausea. Diffuse abdominal pain is also present if rupture and intraperitoneal bleeding occurs. Given that Luanne is two weeks from her last menstrual period, she reports no vaginal bleeding, and her pain is intense and located in the middle of her abdomen, an ectopic pregnancy is less likely. D. Appendicitis is the most common condition in children requiring immediate surgical intervention, but often (especially in infants) presents differently than in adults. Aspects of their atypical presentation include lack of migration of pain to the RLQ, negative Rovsing's sign, and involuntary guarding and fever without perforation. School-age children who can articulate the pain often describe pain with movement or coughing (cat's eye sign). Also, rebound tenderness was found to be neither sensitive nor specific in the pediatric population, while in the adult population it is one of the most accurate PE findings (86%). Luanne is of the older pediatric population, and so will present with a more typical appendicitis. Her sudden onset of intense pain at the umbilicus with vomiting, anorexia, and tenderness at McBurney's point are all classic findings. The more atypical signs include diffuse pain centered below the umbilicus, and rebound tenderness that might point to a perforation (more likely, it is part of the atypical pediatric presentation given her normal WBC). Another atypical aspect of her exam is her adnexal pain during the pelvic exam, which could be due to the degree of inflammation and the positioning of her appendix. The key take-away point is to have a high index of suspicion for appendicitis in pediatric patients with abdominal pain given their atypical presentation.

A 4-year-old girl with a history of type 1 diabetes mellitus was admitted to a local hospital for treatment of DKA. A few hours after the treatment, she develops grunting, irregular respirations, and has vomited twice. On exam, her left eye is pointing downward and out on straight gaze. Her diastolic blood pressure is 90 mmHg. What is a likely diagnosis? A. Hypoglycemia B. Hypokalemia C. Hyponatremia D. Pneumonia with possible sepsis E. Cerebral edema

16.2 > E has been selected by the expert. A. This choice is incorrect because although hypoglycemia can be a complication of DKA treatment, it would be unlikely to cause a cranial nerve palsy. B. This choice is incorrect. Although hypokalemia can be a complication of DKA treatment, it would likely present as an elevated BP, muscular weakness or myalgia, as well as muscle cramps, constipation, and hyporeflexia in severe cases, rather than with the symptoms described in this vignette. C. This choice is incorrect because, while hyponatremia can occur (due to the dilutional effect caused by water shifting from the intracellular to the extracellular compartment because of hyperglycemia and increased plasma osmolarity), it would be corrected with the DKA treatment and would not present with a cranial nerve palsy. D. This choice is incorrect because pneumonia would not present with a cranial nerve palsy, although it might be important to evaluate patients with DKA for signs of intercurrent illness, including pneumonia, UTI, and perinephric abscess. E. This choice is correct. Administration of bicarbonate during DKA treatment increases the risk of cerebral edema. Although symptomatic cerebral edema is rare (less than 1%), it is associated with a high mortality rate (over 20%). The signs of cerebral edema are described in the vignette, and include irregular respirations, headache, vomiting, third nerve palsy, and high blood pressure.

A 9-year-old female is brought to clinic by her mother because of two days of abdominal pain and vomiting. She has vomited six times today and has had decreased appetite, but no diarrhea, fevers, sick contacts, or changes in diet. Her mom states that she has been otherwise healthy apart from increased thirst and occasional bedwetting over the last few weeks. Of note, patient's maternal grandmother suffers from celiac disease. On exam, patient is afebrile and has a HR of 180 bpm, BP 90/60 mmHg, RR 50 bpm, and O2 saturation of 98%. She is lying in bed appearing slightly drowsy, taking rapid, deep breaths and is slow to respond to questions. Her heart and lung exams are normal apart from being tachycardic, and abdominal exam reveals mild diffuse tenderness to palpation with no rebound or guarding. Which of the following would be the most appropriate next step in management? A. Chest x-ray B. Urine culture C. Fingerstick glucose D. Abdominal ultrasound E. Gastric lavage

16.3 > C has been selected by the expert. A. A chest x-ray would be appropriate if bacterial pneumonia were high on your differential. A patient with pneumonia generally presents with fever, cough, tachypnea, and will likely have characteristic lung findings such as crackles on exam. The patient may experience abdominal pain secondary to pleural inflammation; however, vomiting is not a common presentation. The absence of fever, cough, sick contacts, and lung findings on exam make pneumonia a less likely diagnosis. B. Urine culture would be appropriate if pyelonephritis were high on your differential. A patient with pyelonephritis may present with history of fever, dysuria, urinary frequency, CVA tenderness, and vomiting. However, this patient's overall clinical picture does not support the diagnosis, since the patient is afebrile without a history of dysuria or classic CVA tenderness. C. Obtaining a fingerstick glucose is the diagnostic step with the highest yield since the patient's clinical picture is strongly indicative of diabetic ketoacidosis (DKA). DKA is a condition more closely associated with type 1 (rather than type 2) diabetes, and is formally diagnosed if a random glucose is > 200 mg/dL, venous pH is < 7.3 and/or bicarbonate < 15 mEq/L and there is ketonemia or ketonuria. Patients in DKA can present with abdominal pain and vomiting secondary to metabolic acidosis that stems from ketonemia and lactic acidosis. Furthermore, osmotic diuresis from hyperglycemia may contribute to dehydration, which can manifest as tachycardia, hypotension, and altered mental status. In an attempt to compensate for the metabolic acidosis, the patient may also present as tachypneic with characteristic Kussmaul respirations (rapid, deep breaths). This patient's history of polydipsia, enuresis, and family history of autoimmune disease (including celiac disease and Hashimoto's thyroiditis) suggest that the patient has type 1 diabetes. Her current vital signs and general state of lethargy also point towards DKA and should be confirmed with a fingerstick glucose (in addition to other tests). D. Abdominal ultrasound would be indicated if appendicitis were high on your differential. Appendicitis may present with fever, nausea, vomiting, diarrhea, decreased appetite, and abdominal pain that localizes to the RLQ. Physical exam findings may include a positive Rovsing's sign or a positive psoas or obturator sign. Ultrasound may be a useful tool in detecting appendicitis in children. Appendicitis is less likely in this patient since she is afebrile and does not have the classic localizing pain characteristic of this condition. E. Gastric lavage may be the indicated next step of management if toxic ingestion is the confirmed diagnosis. A patient who has ingested toxic substance is usually afebrile and obtunded and may present with vomiting and manifestations of dehydration (secondary to vomiting). In a patient with salicylate toxicity, tachypnea is a common presentation. A thorough history to assess exposure to toxins and sending urine for evaluation of toxins may be appropriate to make this diagnosis. Given this patient's overall picture, DKA is a more likely diagnosis.

A 9-year-old boy presents to the ED in an ambulance after he was found unconscious at a local playground. In the ED he is arousable but extremely obtunded. He is able to minimally verbalize that his head hurts and his stomach feels uncomfortable. He states the pain is constant and non-radiating. He vomits clear liquid twice over the course of 30 minutes. Vital signs are as follows: T 37.6 C, P 66 bpm, BP 155/80 mm Hg, RR 18 bpm. You further notice that his breathing is irregular with brief episodes of apnea. On physical exam you are unable to reproduce the abdominal pain and there is no rebound tenderness or guarding. The rest of the physical exam is unremarkable. What is the most likely diagnosis? A. DKA B. Appendicitis C. Intracranial hemorrhage D. Gastroenteritis E. Small bowel obstruction

16.4 > C has been selected by the expert. A. Choice A is incorrect, because in a patient with DKA one would expect increased adrenergic tone leading to tachycardia, not inappropriate slowing of the HR. Secondly, the patient's breathing pattern is more consistent with Cheyne-Stokes respirations, not Kussmaul breathing. Kussmaul breathing is typically characterized by deep breaths that may be rapid, normal or slow in rate without periods of apnea, often associated with metabolic acidosis. Lastly, one would expect signs or symptoms consistent with dehydration such as polyuria, polydipsia, decreased skin turgor, or skin tenting. However, the altered mental status, vomiting, headache, and abdominal pain could be seen in DKA. B. Choice B is incorrect because the lack of fever, inability to reproduce the abdominal pain in the RLQ, and the severely altered mental status argue against the diagnosis. In appendicitis, children will often complain of a migratory pain that beings around the periumbilical region and migrates to the RLQ. Patients often complain of rebound tenderness and demonstrate guarding as well. C. This is the correct choice. Increased ICP can be secondary to epidural or subdural hemorrhage. It is possible the patient may have fallen while playing in the playground. Increased ICP can present as the classic Cushing's triad: hypertension, inappropriate slowing of the heart rate, and irregular respirations (Cheyne-Stokes respiration). A further complication of increased ICP is epigastric discomfort. This is caused by the elevated ICP causing vagal stimulation, resulting in the secretion of gastric acid. Lastly, the patient's headache and non-bilious vomiting can also be ascribed to the increased ICP. D. Choice D is incorrect because gastroenteritis usually presents with fever, colicky abdominal pain, and diarrhea. It would also be atypical for the patient's mental status to be so adversely affected by gastroenteritis. More than 95% of gastroenteritis hospitalizations occur in children younger than 5 years, with the peak incidence between 3 and 24 months of age. The incidence tends to peak in winter. There can be both viral and bacterial causes for gastroenteritis. Classically, viral gastroenteritis will present with diarrhea in which the stool lacks blood or mucus. Bacterial gastroenteritis often causes diarrhea with gross blood or mucus present in the stool. E. Choice E is incorrect because a small bowel obstruction usually presents with bilious vomiting, abdominal distention, inability to pass flatulence, and moderate-to-severe abdominal pain. The pain is often paroxysmal, coming and going in 4 to 5 minute intervals. The patient's lack of a fever, however, is consistent with a GI obstruction. The most common causes of a small bowel obstruction are adhesions from a previous surgery or a hernia.

A 7-year-old boy is brought by ambulance to the ED with altered consciousness. The EMT said he found the boy in a pool of vomit. He is unable to answer questions coherently and he is alone. Physical exam findings indicate dry mucous membranes, tachypnea, tachycardia, and moaning on palpation of the abdomen. His physical exam is otherwise normal, including a normal blood pressure. What is the most likely cause of his condition? A. Appendicitis B. DKA C. Narcotic overdose D. Non-accidental trauma E. Sepsis

16.5 > B has been selected by the expert. A. Appendicitis would rarely present with altered consciousness. The usual history with appendicitis is onset of periumbilical pain that persists over hours, migrating to the right lower quadrant. Vomiting could be present, and tachycardia may be present due to pain or dehydration, but altered mental status would be unusual. On physical exam, peritoneal signs may be present as well as psoas, obturator, or Rovsing's sign. B. DKA typically presents with altered mentation, vomiting, dehydration, and abdominal pain. The history will yield polydipsia and polyuria during the days preceding DKA. Metabolic acidosis causes tachypnea as the body tries to blow off CO2 through a compensatory respiratory alkalosis. C. Although he may have access to narcotics, his presentation does not fit well with this choice. Signs of narcotic overdose include pinpoint pupils, depressed respiratory rate, and altered consciousness. His tachypnea and lack of pinpoint pupils argue against this choice. D. Lack of fractures, bruises, or burns argues against this choice. Trauma resulting in increased intracranial pressure may result in hypertension, bradycardia, and disordered breathing. E. Sepsis can present with altered mental status. This child's presentation is less consistent with sepsis given that he doesn't have fever or other vital sign changes consistent with sepsis syndrome (temperature > 38.5°C or < 36°C, hypotension, along with warm, dry extremities).

A 9-year-old boy is brought to the ED in a coma secondary to diabetic ketoacidosis. Which of the following laboratory results would NOT likely be found in this child? A. Anion gap of 20 mEq/L B. Potassium of 3.3 mEq/L C. Venous pH of 7.1 D. Sodium of 132 mEq/L E. Creatinine of 1.0 mEq/L

16.6 > B has been selected by the expert. A. DKA causes a metabolic acidosis from the elevated level of ketones. The elevated level of ketoacids and lactic acid requires buffering by bicarbonate, thus leading to an increased anion gap. B. In diabetic ketoacidosis, the acidosis and lack of insulin cause potassium to leave cells and enter the serum, causing an elevated serum potassium level. However, as the DKA is corrected and insulin is administered, the potassium will re-enter the cells, causing a decreased serum potassium level, so potassium levels should be monitored closely when therapy is initiated. C. The pH would be low due to the metabolic acidosis caused by the elevated level of ketones. D. Hyponatremia is seen in DKA because the hyperosmolarity of the intravascular space from the increased glucose levels causes osmotic movement into the extracellular space. Additionally, there is increased sodium loss from the kidneys. A corrected sodium level should be calculated to adjust for the hyperglycemia, using the following formula: corrected sodium = [{(measured glucose - 100)/100} x 1.6] + measured sodium. E. Patients with DKA are often dehydrated when they present to the ED. This causes a prerenal azotemia, which presents as an elevated creatinine level.

An 8-year-old obese boy comes to the clinic with a chief complaint of right knee pain with the right foot medially rotated. On an exam the right knee is neither swollen nor erythematous but he is noted to have a limited ROM of the right hip. In addition, when he lifts his right leg, it externally rotates. The patient did not have a URI or any trauma preceding the onset of pain. The vital signs are normal at the time of the visit and he is well appearing and afebrile. What is/are the best next step(s) in management? A. AP and lateral x-ray followed by casting and crutches B. Bone scan C. AP and lateral x-ray followed up by internal reduction of the femoral head D. Aspiration of the knee E. Observation and weight reduction counseling

17.1 > C has been selected by the expert. A. This choice would be correct if we suspected Legg-Calves-Perthes disease, or avascular necrosis of the capital femoral epiphysis. But the knee pain and physical exam findings are not consistent with this diagnosis. B. This choice is incorrect because the bone scan is used to diagnose an acute hematogenous osteomyelitis (although MRI is a more commonly used tool). This child is not febrile and has no localized bone pain and is therefore less likely to have osteomyelitis. C. This choice is correct because the AP and lateral x-rays are needed to diagnose a slipped capital femoral epiphysis, which is considered an emergency. This patient's age group, his obesity, and the description of the external rotation of the right leg when the hip is flexed all suggest this diagnosis. D. This choice is incorrect because the aspiration and drainage via arthroscopy is done for a septic joint. Based on the physical exam and vital signs, the patient is unlikely to have an infection of the bone or joint. E. This choice is incorrect because observation is not optimal when the patient is likely to have an urgent condition that needs to be fixed. Weight reduction counseling might be a good option after his problem is addressed.

A 6-year-old girl comes to the clinic because of worsening right knee pain over the past month. On exam, you note generalized lymphadenopathy and splenomegaly. She coughs intermittently throughout the visit, and her mother explains that she is just getting over a cold. You note absence of tenderness, erythema, effusion or warmth over the hip, knee, or ankle joints. Her vitals are unremarkable except for a low-grade fever (100.8 F). Reviewing her chart, you note that she has lost 5 lbs since her visit 2 months ago. She sits with her right leg externally rotated but appears to be in pain despite trying several different positions, refusing to bear weight on that side. What is the most likely diagnosis? A. Reactive arthritis B. Leukemia C. Osteomyelitis D. Transient synovitis E. Septic arthritis

17.2 > B has been selected by the expert. A. Choice A is incorrect because this patient is showing systemic symptoms such as weight loss, hepatosplenomegaly, and generalized LAD, not typically seen in reactive arthritis. Reactive arthritis typically follows a viral infection, presenting two to four weeks following the infection. Children are commonly afebrile, and pain may involve multiple joints. B. Choice B is correct. Leukemia can present as bone pain due to replacement of bone marrow by leukemic cells. Patients may present with a limp or refusal to walk. Leukemia is associated with systemic symptoms such as low-grade fever, chronic/insidious joint pain, generalized LAD, weight loss, and/or hepatosplenomegaly. C. Choice C is incorrect because osteomyelitis typically presents with point tenderness over the bony joint and signs of joint inflammation. Osteomyelitis is most commonly associated with infection by Staph aureus or Strep pyogenes. The pain is worse upon weight-bearing, and fever is seen in about 50% of cases. D. Choice D is incorrect because this patient is showing systemic symptoms such as weight loss, hepatosplenomegaly, and generalized LAD. Transient synovitis would be high on the differential if this child were otherwise well-appearing with isolated involvement of the joint, most commonly the hip. Transient synovitis may follow an upper respiratory infection and usually resolves on its own within three to seven days. E. Choice E is incorrect because the history and physical are more suggestive of leukemia; however, septic arthritis may present similarly. Erythema, warmth, and swelling of a deep joint may not be readily apparent on exam. While fever is strongly associated with septic arthritis, it would likely be higher (> 38.5 °C) and the patient would have a more acute presentation. A CBC with differential would be useful in this situation. An ultrasound of the joint would also be able to identify an effusion and/or guide joint aspiration if septic arthritis was suspected. Most common organisms responsible for septic arthritis by age include: Staph aureus, GBS, E coli (neonates < 2 months); Staph aureus, Haemophilus influenzae, Strep pneumo (older children); Neisseria gonorrhea (adolescents).

A 3-year-old girl comes to the clinic with a limp and a slightly externally rotated right hip. Which of the following signs/symptoms would you expect in the history or exam if a diagnosis of transient synovitis were made? A. History of a recent upper respiratory tract infection B. High-grade fever C. Iridocyclitis D. Knee pain E. ESR of 110 mm/hr

17.3 > A has been selected by the expert. A. Transient synovitis of the hip is associated with a low-grade fever and frequently occurs during or after a URI. Between 32% and 50% of children who present with transient synovitis had a recent upper respiratory tract infection. It is also important to remember that transient synovitis is a diagnosis of exclusion, and it is important to rule out other causes of hip pain that may require urgent intervention, such as septic arthritis. B. High-grade fever would be more concerning for osteomyelitis or septic arthritis, and would not be expected in transient synovitis (although low-grade fever is possible). However, fever occurs in only half of cases of osteomyelitis. Other symptoms associated with osteomyelitis include localized bony tenderness, indolent presentation, and refusal to bear weight. Osteomyelitis cannot be ruled out due to absence of fever. C. Iridocyclitis is associated with juvenile idiopathic arthritis (JIA). Other findings associated with JIA include fever and rash. Diagnostic criteria for JIA include children less than 16 years old and at least 6 weeks of arthritis in at least one joint. D. Hip pathology can present as knee pain. However, it is not something that you would use to rule in or rule out transient synovitis (it is not specific for transient synovitis). E. A significantly elevated ESR would be concerning for septic arthritis. In fact, in one study, an ESR > 40 mm/hr was one of the criteria found to be more associated with septic arthritis than with transient synovitis. Other criteria associated more with septic arthritis than with transient synovitis include: refusal to bear weight, erythema, warmth, swelling, elevated CRP, and an elevated WBC count.

A 3-year-old girl is at the pediatrician's office for continued right knee pain after a ground-level fall six weeks ago. The patient is UTD on all immunizations, has no significant PMH, and no recent illnesses. The patient's mother reports the patient complains of pain mostly in the morning when going to daycare but doesn't seem to be bothered by it while playing outside in the afternoon. On exam the patient's vitals are all within normal limits. Her physical exam reveals a well-appearing toddler who walks stiffly and avoids bending her right knee. The knee has a mild effusion but no obvious erythema. There is pain with passive flexion and extension of the right knee. During the exam the girl tells you her left ankle also hurts, which her mother had forgotten about but says started hurting the same time as the right knee. Her CBC is normal, while her ESR and CRP are mildly elevated. Which of the following is the most likely cause of this child's condition? A. Septic arthritis B. Leukemia C. Juvenile idiopathic arthritis D. Transient synovitis of the hip E. Bacterial osteomyelitis

17.4 > C has been selected by the expert. A. Septic arthritis is incorrect because the patient has been generally well for the past six weeks and presents with mild swelling but no erythema of her joint. She is afebrile with no leukocytosis on CBC. While each of these alone would not rule out septic arthritis, the combination of all these factors together make septic arthritis unlikely. Patients with septic arthritis are often febrile. Joint fluid analysis often reveals elevated WBCs and predominant neutrophils (although a joint tap in this scenario is not necessary based on the presenting symptoms and exam). Bacteria may also be cultured from the joint fluid. Additionally, her CRP and ESR are only mildly elevated. One would expect to see a greater elevation of these markers of inflammation in septic arthritis. B. Leukemia is incorrect because the patient does not present with other systemic symptoms such as fever, weight loss, lymphadenopathy, hepatosplenomegaly, petechiae, or bruising. This patient also has pain that improves with activity, as opposed to a more chronic pain that does not improve with position or movement that would be expected with leukemia. Additionally, a normal CBC without thrombocytopenia or anemia makes this answer unlikely. C. Juvenile idopathic arthritis (JIA) is correct for several reasons. Pauciarticular juvenile arthritis is the most common type of JIA (60% of JIA) and causes pain in four or fewer joints for six or more weeks. This patient is generally well even after six weeks of pain, which would be unlikely if this patient had septic arthritis. Her pain improves with activity, and the ESR/CRP are only mildly elevated. On exam, she has a mild effusion but no obvious erythema. In cases of systemic JIA, patients may have a rash which lasts only a few hours (evanescent) that is also macular and salmon-colored, and high-spiking and appears periodically (once or twice a day); however, this form of JIA is not consistent with this patient's history. D. Transient synovitis of the hip is incorrect because this condition, while acute, generally resolves in three to four days and this patient has had pain for six weeks. Transient synovitis typically affects the hip, but patients may also report knee or inner thigh pain. Commonly, patients may have a history of a recent URI. Additionally, patients with transient synovitis do not typically present with a joint effusion as is seen in this patient. E. Bacterial osteomyelitis in children, usually hematogenous in origin, presents with acute onset of bone pain and fever and may involve a contiguous joint. This patient is well appearing and has no leg tenderness.

A 4-year-old child is refusing to walk over the course of a week. Her mother recalls that she fell off her bike yesterday. On exam, she is afebrile, but has decreased ROM of her hip. You review her file and note that she is up-to-date on her immunizations and she was last seen three weeks ago for a self-limited episode of diarrhea that she developed while visiting family in rural Mexico. Aspiration of her affected hip joint reveals slight increase in inflammatory cells but normal chemistries and a negative gram stain. Culture is pending. Which of the following is the most likely diagnosis? A. Osteomyelitis B. Trauma C. Transient synovitis D. Reactive arthritis E. Septic arthritis

17.5 > D has been selected by the expert. A. Incorrect. The gradual onset of refusing to walk is consistent with osteomyelitis, and only half of patients present with fever. However, this patient is not acutely ill as one would expect with hematogenously acquired osteomyelitis. While this is a possible diagnosis, it is further down on the differential diagnosis. B. The timing of her pain is not consistent with trauma, since her pain started one week ago, but she fell off her bike yesterday. Of note, children who present with limp secondary to trauma may not have any findings visible on x-ray for a few weeks. C. While transient synovitis could certainly present with pain, it is more likely to present acutely in the absence of other complaints. A recent URI, not gastroenteritis, is most consistent with a transient synovitis. D. Correct. The patient likely had a recent case of mild to moderate gastroenteritis in Mexico, which may have been secondary to an bacterial enteritis such as shigella, or campylobacter. In reactive arthritis, joint inflammation occurs a few weeks later because antibodies made during the illness are attacking the joint. While several inflammatory cells would be seen in the aspirate, importantly, the cultures will turn out to be negative. E. Although on exam it may be difficult to distinguish septic from reactive arthritis, in septic arthritis the culture is often positive. These patients usually have acute onset of pain, fever, and limp, and the hip or knee is commonly affected. It is caused by an infection, typically bacterial, in the joint space. While definitely high on the differential for this patient, it is not the best answer choice given the history of diarrhea and the benign finding on joint aspiration.

A 5-week-old infant is brought to the pediatrician for failure to thrive (despite adequate, even prolonged, feedings) and respiratory distress (particularly tachypnea). What other features does this infant most likely have? A. Cyanosis from a right-to-left shunt B. Systolic murmur with a widely split second heart sound C. An early systolic click without a murmur D. A hyperdynamic precordium with a holosystolic murmur

18.1 > D has been selected by the expert. A. Incorrect. Cyanosis from a right to left shunt would occur in the setting of either cyanotic congenital heart disease (such as tetralogy ofFallot) or an unrepaired left to right shunt such as a VSD. Children with cyanotic heart defects typically present with cyanosis, not tachypnea and poor weight gain. An unrepaired left to right shunt such as a VSD can eventually lead to an increase in pulmonary vascular resistance, which is referred to as Eisenmenger's syndrome. When this occurs, the pulmonary vascular resistance equals and then exceeds that of the systemic vascular resistance, leading to reversal of the direction of blood flow through the VSD to become a right-to-left shunt. However, this generally occurs if the VSD is allowed to persist for months to years, which is less likely in our 5-week-old infant. B. Incorrect. A widely split, fixed S2 indicates an atrial septal defect (ASD). These are often detected in children when they're between 3 and 5 years old. The systolic murmur is due to the increased blood flow across the pulmonic valve. The widely split fixed second heart sound indicates an ASD is the cause of the murmur rather than an innocent heart murmur. C. Incorrect. An early systolic click occurs when an abnormal semilunar valve opens at the end of isovolumic contraction early in systole. This occurs when the semilunar valve (aortic or pulmonic) is bicuspid. If the bicuspid valve is also stenotic, the click will be accompanied by a systolic murmur; if the bicuspid valve is regurgitant, the click will be accompanied y a diastolic murmur. A click without a murmur implies although the valve is anatomically abnormal, its function is normal. Thus a bicuspid semilunar valve that is not accompanied by a murmur has normal function and would not be expected to contribute to symptoms in infancy. D. Correct. A heart murmur from a VSD is typically not appreciated in the immediate newborn period, as the pulmonary vascular resistance is still quite elevated. During this time, since the pulmonary vascular resistance equals the systemic vascular resistance, there is no shunting of blood through the open VSD. However, after a few days to weeks after birth, the pulmonary vascular resistance decreases, and the murmur appears, reflecting the shunted flow of blood through the open VSD (from left to right). As flow through the VSD increases and becomes audible, the child is at risk for developing symptoms related to excessive pulmonary flow: tachypnea and poor weight gain.

You have accepted a part-time tutoring job for first-year medical students. One of your students asks if you would please clarify the details of normal fetal circulation. Which of the following best describes the path oxygenated blood takes to reach the fetal brain? A. RA > foramen ovale > LA > LV > systemic circulation B. RA > RV > VSD > LV > systemic circulation C. RA > RV > pulmonary circulation > LA > LV > systemic circulation D. RA > RV > ductus arteriosus > LV > systemic circulatio E. RA > RV > ductus arteriosus > systemic circulation

18.2 > A has been selected by the expert. A. Correct. In fetal circulation, the foramen ovale connects the RA to the LA, allowing a portion of the blood to bypass the RV and the lungs. Approximately a third of the blood that enters the RA passes through this route (preferentially the most oxygenated via the unbilical veins which joined the IVC to reach the RA, which is then delivered to the brain and heart), leaving the majority of the blood to travel into the RV. Closure of the foramen ovale is a normal transition from fetal to extrauterine circulation. B. Incorrect. VSDs are common congenital heart defects, and are not considered a part of normal fetal circulation. C. Incorrect. In utero, without ventilation, the pulmonary vasculature is a high-resistance system. As such, only 8 to 10% of the blood that enters the RV flows through the circulation. D. Incorrect. The ductus arteriosus does not empty into the LV, but rather into the descending aorta. E. Incorrect. The majority of the fetal circulation travels this route. Approximately 90-92% of the blood that enters the RV (two-thirds of the blood that enters the RA) travels out and through the ductus arteriosus, bypassing the pulmonary circulation and the left heart, ending up in the descending aorta. This blood is preferentially less oxygenated than that which flows through the foramen ovale. Like the foramen ovale, closure of this bypass is a normal transition from intra to extrauterine life.

A 5-year-old boy is noted to have a grade II systolic murmur and a widely split S2 murmur on cardiac exam. His vital signs are stable and he has been asymptomatic. Which of the following statement is accurate regarding this child's presentation and likely condition? A. No further work-up for a presumed venous hum B. Chest x-ray, ECG, and echocardiogram would be indicated as next steps to work up a presumed ventricular septal defect C. This patient's murmur is caused by flow through the pulmonary outflow tract and should be evaluated D. The patient should be scheduled now for cardiac catheterization

18.3 > C has been selected by the expert. A. Incorrect. A benign venous hum would not be expected to present with a widely split second heart sound. B. Incorrect. The first steps in the evaluation of presumed structural heart disease would include a chest x-ray, ECG, and echocardiogram. However, the clinical presentation is not consistent with a ventricular septal defect, which usually presents with a holosystolic murmur. C. Correct. This patient's murmur is likely caused by an atrial septal defect, which causes flow of additional blood through the pulmonary outflow tract and should be evaluated. D. Cardiac catheterization would not be indicated as first line evaluation. Noninvasive testing such as an ECG and echocardiogram would be indicated before considering a cardiac catheterization for diagnostic/therapeutic intervention.

A 6-week-old African-American infant presents to your office for a check-up. The baby was born at term by NSVD to a 29-year-old G1P0 mother with no complications. Mother states the baby was feeding well until a week ago, when he developed increased sleepiness, prolonged feeding, and greater duration between feeds. His mother notes he stops to take breaks sometimes because he seems to be trying to catch his breath. He has 4 to 6 wet diapers per day and poopy diapers 3 or 4 times per day. Vital signs are: T: 37.6 C, RR: 68 bpm, P: 138 bpm, BP: 88/58 mmHg, and 02 saturation is 98%. The physical examination is notable for increased respiratory effort and retractions, and, upon cardiac examination, a murmur with a hyperactive precordium and no cyanosis. Abdominal exam reveals a liver edge palpable to 4 cm below the right costal margin. Which condition would be *least* likely to be the cause of the infant's symptoms? A. Aortic stenosis B. Coarctation of the aorta C. Ventricular septal defect D. Patent ductus arteriosus E. Atrial septal defect

18.4 > E has been selected by the expert. A. Choice A is incorrect because aortic stenosis is one of the heart defects that can present with a murmur and signs of congestive heart failure in infancy. An estimated 10-15% of patients with aortic valve stenosis present when they are younger than one year of age. Neonates with critical stenosis are typically symptomatic and present with symptoms of congestive heart failure-including poor feeding, rapid breathing, poor urine output, and fussiness-as the ductus arteriosus closes and systemic blood flow decreases. B. Choice B is incorrect because coarctation of the aorta is one of the heart defects that can present with a murmur and signs of congestive heart failure in infancy. Pediatric patients may present in the first few weeks of life with poor feeding, tachypnea, and lethargy and progress to overt CHF and shock. Symptoms may be subtle at first, and patients may make repeated trips to the physician before being diagnosed. Presentation after the neonatal period is usually consistent with hypertension or a murmur. Older patients usually have not developed overt CHF because of the presence of arterial collateral vessels. C. Choice C is incorrect because ventricular septal defect is one of the heart defects that can present with a murmur and signs of congestive heart failure in infancy. Signs and symptoms often appear during the first few days, weeks, or months of a child's life. VSD is an acyanotic congenital heart defect, manifesting as a left-to-right shunt. A holosystolic murmur is often appreciated, with larger VSDs causing a parasternal heave. An infant with a large VSD will fail to thrive and become diaphoretic and tachypneic, especially with feeding. D. Choice D is incorrect because a large patent ducutus arteriosus (PDA) is one of the heart defects that can present with a murmur and signs of congestive heart failure in infancy. A PDA is more common in premature infants and those with neonatal respiratory distress syndrome. A smaller PDA may not cause any symptoms, but infants can present with tachypnea, poor feeding, tachycardia, shortness of breath, fatigue, diaphoresis, and poor growth. E. Choice E is correct because atrial septal defects (ASDs) do not cause CHF. ASDs often go undiagnosed for decades due to subtle physical examination findings and/or a lack of appreciable symptoms. Children with ASD's are generally asymptomatic.

A 3-year-old boy is brought to the clinic by his parents for follow-up of iron deficiency anemia. His diet consists mostly of sweet, bland, low-texture foods. He drinks 32 ounces daily of milk from a bottle. In addition to prescribing oral iron supplementation, what is the best advice to give the parents concerning this patient's diet? A. Continue bottle-feeding B. Encourage eating small amounts of food throughout the day C. Gradually introduce new foods and slowly decrease his old favorites D. Bribe the patient to eat healthy food E. No change is needed since he is on the optimal diet for his age

3.1 > C has been selected by the expert. A. This choice is incorrect since children should be weaned from a bottle at their first birthday. B. This choice is incorrect because the child should be encouraged to restrict eating to 3 meals and 2 snacks per day, instead of "grazing" throughout the day. C. This choice is correct, because gradually introducing new foods and slowly decreasing his old foods will likely ease the transition to healthier diet choices and encourage long-term adjustment. D. This choice is incorrect because bribing is unlikely to be effective. He should be presented with healthy options only, and dessert should not be used as an incentive for healthy eating. E. This choice is incorrect because the child's current diet is not optimal. He should be eating a varied diet with the recommended servings of fruits and vegetables per day.

A 2-year-old girl is brought to the clinic by her mother for a health maintenance visit. While waiting for the pediatrician, her mother reads her a short book. When you enter the room, her mother asks her to take the book and return it to a bookshelf. Developmental history reveals that the patient can scribble with a pencil but cannot write her name. She can kick and throw a ball, but cannot jump in place. Which of the following best describes this child's development? A. Delayed language B. Delayed social skills C. Advanced fine motor skills D. Advanced gross motor skills E. Age-appropriate development

3.2 > E has been selected by the expert. A. Delayed language is incorrect: A 24-month-old child is expected to use pronouns inappropriately, but should be able to follow two-step commands such as taking a book and returning it to a location in the room. B. Delayed social skills is incorrect: At 24 months of age, children are able not only to listen to short stories, they also engage in parallel play. C. Advanced fine motor skills is incorrect: While a child can hold a pencil at 24 months, the grip is immature and the child imitates pencil strokes. Children can remove their pants and socks at this age, but need help to undress completely. D. Advanced gross motor is incorrect: Being able to jump in place is a 30-month-old milestone. Being able to throw a ball overhand is expected at 24 months of age. E. The child in this vignette is developmentally appropriate for her age.

A 4-year-old boy is brought to the clinic by his mother for developmental evaluation. She is concerned that he is delayed when compared to the children of her friends. Although he can throw a ball and copy a circle, he cannot brush his teeth on his own, tie his shoes, or hop on one foot. Which of the following are the expected milestones in this patient? A. Throw a ball overhand, ride tricycle, build tower of 6-8 cubes B. Hop on 1 foot, copy a cross, brush teeth C. Tie a knot, copy squares D. Mature pencil grasp, print some letters and numbers E. Skip, draw a person with 6 or more body parts

3.3 > B has been selected by the expert. A. This choice is incorrect. Throwing a ball overhand, riding a tricycle, building a tower of 6-8 cubes, and copying a circle are developmental milestones for 3-year-olds. B. This choice is correct. A a normally developing 4-year-old should be able to hop on 1 foot, copy a cross, pour/cut/mash their own food, and brush teeth. C. This choice is incorrect. These are milestones for 5-year-olds. D. This choice is incorrect. These are milestones for 5-year-olds. E. This choice is incorrect. These are milestones for 5-year-olds.

A 3-year-old boy described by his mother as a picky eater comes in for a regularly scheduled well-child visit. His mother complains that he has had less energy than usual for the past few months. There is a high clinical suspicion he is anemic. Which of the following is most correct? A. Complete blood count B. Serum lead C. Serum iron D. Serum folate E. Albumin

3.4 > C has been selected by the expert. A. This choice is incorrect. This child is at risk of iron deficiency anemia. Although a CBC may show microcytic anemia, a serum iron level is more specific. B. This choice is incorrect. Most 3-year-old children no longer mouthing objects. Although they may have been exposed to lead in the past one would explore other options of fatigue first. Lead levels are usually checked at 1 and 2 years of age. C. The most likely cause of anemia in the question is picky eating, which can result in insufficient iron intake. Low iron intake causes a microcytic anemia. D. This choice is incorrect. Iron deficiency is the most common cause of anemia in this scenario. Note that folate deficiency may be associated with a goat's milk diet. E. This choice is incorrect. Protein malnutrition is uncommon in children in developed countries.

A 5-year-old girl is brought by her mother to the clinic for a well-child visit. Medical history is significant for 3 upper respiratory infections in the past year. She does well in preschool, is toilet trained, and enjoys eating mostly pasta, bread, and milk. She lives with her mother and father in a home built in 1985. Her height is at the 50%, weight is at the 50% and BMI is at the 60%. Vital signs are normal. Physical examination is normal. Lab studies today show a hemoglobin of 10.0 g/dL. Her hemoglobin was in the normal range at her 3-year-old visit. Which of the following is the most likely cause of her anemia? A. Chronic blood loss B. Lead poisoning C. Chronic illness D. Iron deficiency E. Hemoglobinopathy

3.5 > D has been selected by the expert. A. This choice is incorrect. There was no complaint of melena, and the child is overall healthy. Chronic blood loss would therefore be unlikely. B. This choice is incorrect. Risk of lead poisoning is increased in patients who live in homes built before the 1950s. Additionally, this patient does not complain of other symptoms suggestive of lead poisoning: weight loss, lethargy, vomiting, and learning difficulties. C. This choice is incorrect. The patient has no significant past medical history, and the review of systems is unremarkable. Chronic illness is unlikely. D. This choice is correct. Given the patient's age and preference for pasta and milk, the most likely cause of anemia would be iron deficiency. Treatment would include oral iron supplementation and increased dietary iron intake. E. This choice is incorrect. If she had a hemoglobinopathy, she would have been expected to have been anemic at her previous visit as well.

An 8-year-old boy is brought to clinic by his parents because they are concerned that he has not been doing his homework. His teacher recently called the parents to say that their son seems distracted in class, constantly interrupts other children when they are speaking, and is very fidgety. When you speak with the boy, he tells you that he did not know about the homework assignments and that he tries hard to pay attention in class. What is the next best step in management? A. Prescribe a stimulant medication for ADHD B. Suggest behavior modification for the child and parenting classes C. Group therapy for the child D. Do nothing, as this child's behavior is normal E. Contact the teacher to find out more about his behavior. Find out more about the child's behavior at home

4.1 > E has been selected by the expert. A. Pharmacotherapy is often used in combination with behavioral modification/group therapy for children diagnosed with ADHD. However, to be diagnosed with ADHD, one must have 6 or more symptoms in 2 or more settings for at least 6 months, and several of these symptoms must be present before the age of 12. The symptoms fall within the three categories of inattention, hyperactivity, and impulsivity. B. Behavior modification for the child and parenting classes for the parents are both used as treatment modalities in ADHD. Parents should be counseled on positive reinforcement, firm non-punitive limit setting, and how to reduce external stimuli. However, this child first requires further evaluation. C. Children with ADHD often learn best in group therapy, particularly social skills and self-esteem. Again, this child first requires further evaluation. D. Many school-aged children are easily distractible, impatient, and hyperactive. It is important to distinguish those who truly have ADHD from those who do not. Again, to have a diagnosis of ADHD, symptoms must be present in two or more settings. E. Contacting the teacher to find out more about the child's behavior at school and learning more about his behavior at home are the best ways to determine if 6 of the symptoms are present in 2 or more settings, which is required to make the diagnosis of ADHD. It also will be important to learn more about other aspects of this child's life, as there are several factors that can lead to acting out (including learning disability, hearing disability, family stress, and abuse).

An 8-year-old healthy obese African American male with no past medical history is found to have a blood pressure of 125/90 mmHg on all four extremities on routine evaluation during an office visit for well-child care. Review of symptoms is negative. A physical exam and screening bloodwork are performed. Both are normal, with the exception of his blood pressure and obesity. What is the most likely diagnosis? A. Primary hypertension B. Renal artery stenosis C. Coarctation of the aorta D. Pheochromocytoma E. Hyperthyroidism

4.2 > A has been selected by the expert. A. The sole physical finding is hypertension. Given the mild hypertension and the patient's age, symptoms are unlikely to be present. Other etiologies should be ruled out, but review of symptoms, physical examination, and laboratory studies do not suggest other etiologies. B. Patients with renal artery stenosis are largely asymptomatic, but as the kidney function deteriorates, they may experience edema and dyspnea. On physical examination, patients will have hypertension (as a consequence of elevated angiotensin II and aldosterone) and possibly abdominal bruits. For laboratory studies, the patient will have elevated serum angiotensin II and serum aldosterone, as the kidneys attempt to compensate for a perceived decrease in glomerular filtration rate. C. Assuming sufficient severity of the coarctation, symptoms include chest pain, cold lower extremities, dizziness, syncope, exercise intolerance, failure to thrive, poor growth, headache, and dyspnea, among others. Distal to the coarctation, pulses will be diminished. Blood pressure will be lower in the lower extremities compared to the upper extremities. Murmurs may also be heart on auscultation. Coarctation of the aorta may also be associated with other congenital heart defects (bicuspid aortic valve in 50% of patients) and chromosomal abnormalities (Turner syndrome). D. Symptoms include headache, diaphoresis, palpitations, tremor, nausea, weakness, anxiety, nervousness, irritability, and weight loss, among other symptoms. On physical examination, patients typically present with tachycardia and severely elevated blood pressure. For laboratory studies, the patient will have elevated urinary VMA, urinary HVA, urinary metanephrines, and serum glucose. Pheochromocytoma may be associated with various syndromes, such as multiple endocrine neoplasia and Von Hippel-Lindau disease. E. Symptoms include heat intolerance, frequent bowel movements, increased appetite, diaphoresis, nervousness, restlessness, weight loss, tremor, hair loss, and palpitations, among others. On examination, patients typically present with tachycardia, hypertension, hyperreflexia, and goiter. The patient will have elevated T3, T4, and serum glucose, and TSH may be depressed or elevated depending on the etiology.

Billy, a 7-year-old boy, presents to the clinic with complaints of headaches and episodes of feeling sweaty and flushed. He also reports that at times he feels as if his heart is racing. Billy was full term, had an uncomplicated birth, and has been otherwise healthy until now. On exam his BP is 120/80 mmHg and is the same in his upper and lower extremities. His weight and height are in the 50th percentile for his age. What is a likely cause of Billy's hypertension? A. Coarctation of the aorta B. Renal vascular disease C. Renal insufficiency due to renal scarring D. Catecholamine excess E. Primary hypertension

4.3 > D has been selected by the expert. A. Coarctation of the aorta should be suspected in a child with elevated BP (usually > 99th percentile), little family history of HTN, and a discrepancy between upper and lower extremity BPs. Some children with coarctation of the aorta may go undetected until presenting with hypertension at a school-age visit. It is important to pay special attention to the femoral pulses and to document BP measurement in a lower extremity. Billy's BPs in his upper and lower extremities are the same, making this diagnosis less likely. B. Umbilical arterial or venous lines as neonate (most often in premies) can predispose a child to renal vascular disease. Billy had an uncomplicated birth and did not go the neonatal ICU, making it less likely that his hypertension is due to renal vascular disease secondary to an umbilical arterial or venous line as a neonate. C. Recurrent urinary tract infections in childhood are one of the leading causes of hypertension and renal insufficiency later in life due to renal scarring following infections. UTIs are more common in girls. Billy does not have a stated history of urinary tract infections, and his palpitations and flushing are not consistent with hypertension secondary to UTIs. D. Catecholamine excess (pheochromocytoma or neuroblastoma) should be suspected in a child who is hypertensive and has episodes of sudden sweating, flushing, or feels that his heart is racing. Billy is exhibiting these signs and a urine catecholamine testing would be appropriate in this case. E. Most hypertension in children over 6 years of age, and in adolescents, is due to primary HTN. Obesity is an important correlate. Billy's flushing and racing heart would not be fully explained by primary hypertension. Also, Billy is an appropriate weight, making this diagnosis less likely.

Jane is an 8-year-old girl who presents to your clinic for follow-up after being hospitalized for status asthmaticus. She has just completed a 10-day course of systemic steroids. Given her history of moderate persistent asthma, her outpatient regimen includes Advair, a combined steroid and bronchodilator. She was also diagnosed with ADHD one year ago and was started on Concerta, 18 gm PO once a day. Her BMI today is at the 83rd percentile for her age, and her blood pressure is at the 98th percentile for her age. What is the most likely cause of her stage I hypertension? A. Obesity B. The blood pressure cuff is too big C. Medications D. Renal insufficiency E. Neurofibromatosis 1

4.4 > C has been selected by the expert. A. While obesity is a risk factor for hypertension, Jane is not overweight. A child is considered overweight when his or her BMI is between the 85th and 95th percentile for age. Obesity is considered > 95th percentile. B. Blood pressure would be falsely decreased if the cuff was too big, and, inversely, falsely elevated if the BP cuff was too small. C. Both steroids and CNS stimulants can cause increases in blood pressure, especially when used in combination. Steroids increase blood pressure by mimicking endogenous cortisol and the sympathetic fight or flight response. Stimulants mimic norepinephrine, stimulating alpha and beta adrenergic receptors, causing an overall increase in blood pressure. D. Renal insufficiency can be a cause of secondary hypertension, but is unlikely in this patient. Risk factors that warrant investigation of renal causes for hypertension include recurrent UTIs, umbilical arterial/venous lines placed while a child was in the ICU, and a family history of renal disease. E. NF-1 can be associated with hypertension as a result of vascular malformations that affect blood supply to the kidneys; however, this child does not present with any signs of NF-1 (café au lait macules, neurofibromas, optic gliomas, lisch nodules).

George is a 7-year-old boy frequently in trouble at school for being disruptive and inappropriately talkative in class, not following directions set by his teacher, and not working well with classmates during group activities. His mother relates that at home George is always on the go, sleeping only 6 to 7 hours a night. He does not follow her rules all the time either, including not doing his homework, and sometimes putting himself in danger by doing things she tells him not to do, such as running away unaccompanied. Which of the following is the most likely diagnosis? A. Bipolar mood disorder B. Anti-social personality disorder C. Conversion disorder D. ADHD E. Rett syndrome

4.5 > D has been selected by the expert. A. Depression may be responsible for the inattention this child exhibits in school. It is not uncommon for childhood depression to lead to bipolar disorder, in which hyperactivity and impulsivity comprise the manic phase of the disorder leading to a decreased need for sleep. However, the symptoms of depression and mania present in separate phases, not concurrently. The mnemonic commonly used for mania is DIGFAST (distractability, irresponsibility, grandiosity, flight of ideas, agitation, sleep decrease, talkativeness). The mnemonic for depression is SIGECAPS (sleep disturbance, loss of interest, guilt, energy loss, concentration impairment, appetite changes, psychomotor retardation, suicidal ideation). B. Symptoms of anti-social personality disorder include inability to conform to societal norms, disregard of the rights of others, and often criminality. These individuals often exhibit impulsiveness due to their lack of consideration of the consequences of their actions. Males are affected more than females. However, a diagnosis of anti-social personality disorder can be made only in individuals older than 15 years, earlier than which a diagnosis of conduct disorder is appropriate. C. Conversion disorder is a subtype of somatoform disorder. Somatoform disorder would be suspected if no cause could be identified for reported physical symptoms. The drive is unconscious on the part of the patient, and symptoms are not intentionally produced or faked. Conversion disorder is more common in adolescents and involves a sudden loss of sensory or motor functioning. When the patient consciously creates physical symptoms this is referred to as factitious disorder. D. ADHD is characterized by the triad of impulsivity, hyperactivity, and inattention. Other symptoms include motor impairment and emotional labiality. ADHD is typically diagnosed before the age of 7 but persists into adulthood. Intelligence is usually normal, but individuals with ADHD commonly perform more poorly academically than would be expected for their IQ. E. Rett syndrome is an X-linked pervasive developmental disorder seen only in females; affected males die in utero or at birth. The characteristic symptoms involve regression of language and development, intellectual disability, ataxia, and hand-wringing. This disorder is typically diagnosed earlier, at about age 1-4.

A 15-year-old girl comes to the clinic because she has felt tired for one month. She has also had frequent nosebleeds while at school and had been bruising easily. When further history is elicited, you find out that menarche was at the age of 9 and her periods have always been heavy and irregular. Her mother and grandmother also have histories of heavy periods and easy bruising. You suspect a bleeding disorder and send off some labs including a CBC, INR, PT, PTT, and a von Willebrand panel to confirm your diagnosis. Your suspicion was correct for the most common type of bleeding disorder. How is this bleeding disorder most commonly inherited? A. Autosomal dominant inheritance B. Autosomal recessive inheritance C. X-linked recessive inheritance D. Mitochondrial inheritance

5.1 > A has been selected by the expert. A. Autosomal dominant (AD) inheritance is the correct choice. In AD disorders males and females are equally affected within each generation. These include conditions such as von Willebrand disease, Marfan syndrome, neurofibromatosis, and Huntington's disease. B. Autosomal recessive (AR) is incorrect. In AR diseases female and male offspring of heterozygote carriers have a ¼ chance of being affected. These include Tay-Sachs disease and cystic fibrosis. C. X-linked recessive (XLR) is incorrect. In XLR disorders males are more commonly affected, while females are the carriers and pass the disorder on to their sons. Males cannot pass the disease on to other males. These include conditions such as Duchenne's muscular dystrophy, hemophilia, and Fragile X syndrome. D. Mitochondrial inheritance is incorrect. In mitochondrial disorders the disease is inherited only from the mother, and usually all children are affected. Affected males cannot pass the disorder on to their offspring. These include mitochondrial myopathies such as Leber's hereditary optic neuropathy and mitochondrial encephalopathy.

A 14-year-old girl presents to your office wondering why she has not had her period yet. Her mother states that she and the patient's grandmother reached menarche at 13 years of age. The patient is concerned she is behind her friends in terms of development. She is doing well in school and has not had developmental problems in the past. On physical examination, her breasts are elevated without a secondary mound, and curly, coarse pubic hair is present on the labia majora in a triangular shape but does not reach the mons pubis. What Tanner stage would you assign this girl? A. Tanner Stage I B. Tanner Stage II C. Tanner Stage III D. Tanner Stage IV E. Tanner Stage V

5.2 > C has been selected by the expert. A. Tanner Stage I breast development consists of no glandular tissue and is prepubertal. Tanner Stage I consists of no pubic hair at all. This is usually around age 10 or younger. B. Tanner Stage II breast development occurs when breast buds form and the areola begins to widen. A small amount of long, downy hair with slight pigmentation appears on the labia majora. This patient's elevated breast buds and pubic hair distribution puts her beyond Tanner Stage II. C. The patient in the vignette is at Tanner Stage III of development. Her breast buds are elevated but do not have the secondary mound characteristic of Tanner Stage IV. Her pubic hair distribution extends more laterally than Stage II but is not adult-like in hair quality and does not extend onto the mons pubis. D. In Tanner Stage IV, breasts are increased in size and elevation and the areola and papilla form a secondary mound that projects from the contour of the rest of the breast, and the pubic hair extends across the mons pubis and spares the medial thighs. E. In Tanner Stage V, breasts reach their adult size and the areola returns to the contour of the surrounding breast while the central papilla remains projecting and the pubic hair extends to the medial surface of the thighs.

A 16-year-old girl presents to clinic complaining of worsening fatigue. Family history is significant for hypothyroidism and heavy periods in the grandmother. Her exam reveals mild tachycardia and oozing around a recent piercing, but is otherwise normal. Labs reveal Hgb 8.5 g/dL, MCV 58, PT 12.5, PTT 44, and low von Willebrand factor activity. Which of the following is the most appropriate treatment for her underlying disorder? A. Blood transfusion and iron supplementation B. Desmopressin C. Factor VIII concentrate D. Cryoprecipitate E. Vitamin K

5.3 > B has been selected by the expert. A. This patient has iron deficiency anemia, as indicated by her hemoglobin level and MCV. Nutritional deficiency is the most common cause of iron deficiency anemia, and in these cases iron supplementation would be appropriate. However, a blood transfusion and supplemental iron for a presumed nutritional deficiency without investigating potential ongoing losses is inappropriate, especially in a patient with oozing blood on exam. B. Von Willebrand disease is the underlying cause of this patient's anemia, as indicated by the low von Willebrand factor activity. This is the most common hereditary bleeding disorder, occurring in roughly 1% of the population. Intranasal or intravenous desmopressin is appropriate treatment for most bleeding problems. Desmopressin works by causing release of von Willebrand factor from vessel endothelial cells. C. Factor VIII concentrate may be used as a treatment for von Willebrand disease in the setting of major trauma or surgery. It also may be used for less common subtypes of the disease that do not respond to desmopressin. However, most cases of von Willebrand disease can be treated with desmopressin, and therefore it is the most appropriate first choice in treating this patient. D. Cryoprecipitate is a blood product containing fibrinogen, factor VIII, factor XIII, and von Willebrand factor. It is most commonly used to replace fibrinogen. It may also be used in certain scenarios for patients with von Willebrand disease. However, it is not as good as desmopressin as a first choice in this scenario. E. Vitamin K is found in leafy green vegetables and in most multivitamins. It is important in the synthesis of clotting factors. It is given clinically as a one-time IM dose for neonates to assist in clotting factor formation until the gut microbiome is sufficient to provide Vitamin K. It is also used for patients who become supratherapeutic on warfarin therapy. It would not help in this case.

A 10-year-old girl comes to the clinic for a well-child exam. Her mom asks about puberty and wants to know in what order she should expect to see normal developmental changes in her daughter. Which of the following sequences is correct? A. breast bud -> pubic hair -> menarche -> growth spurt B. pubic hair -> breast bud -> growth spurt -> menarche C. pubic hair -> menarche -> breast bud -> growth spurt D. breast bud -> pubic hair -> growth spurt -> menarche E. pubic hair -> breast bud -> menarche -> growth spurt

5.4 > D has been selected by the expert. A. This choice is incorrect because a growth spurt occurs before menarche. B. This choice is incorrect because development of a breast bud is the first sign, followed by pubic hair. C. This choice is incorrect because pubic hair is not the first sign. D. This choice is correct because breast buds are the first sign (10-11 years), followed by pubic hair (10-11 years), then a growth spurt (12 years), and then menarche (12-13 years). Most girls reach adult height by approximately 15 years. E. This choice is incorrect because the order is wrong. The correct order is: breast bud -> pubic hair -> growth spurt -> menarche.

Frank is a 16-year-old boy brought in by his mother who complains that her son "looks much younger than his age." She states that until about four years ago, she did not notice much difference between Frank and his friends. However, in the past two years, Frank has become the shortest person in his class. Frank's mother is concerned that he has a "hormone problem" and wants to know how she can tell if he has begun puberty. What is usually the first sign of puberty in a male? A. Growth of the penis B. Appearance of pubic hair C. Testicular enlargement D. Growth spurt E. First ejaculations

5.5 > C has been selected by the expert. A. Growth of the penis occurs with growth of the scrotum, and usually occurs around 13-14 years of age. It normally follows pubic hair appearance and precedes first ejaculations. B. Pubic hair appearance occurs around 12 years of age, and is usually the second sign of puberty following testicular enlargement. It precedes growth of the penis and scrotum. C. The first sign of puberty in a boy is testicular enlargement. The onset of puberty is quite variable, but usually occurs between 10 and 15 years for boys. It is rare for boys not to have begun puberty by the age of 16. To assess whether or not a male has entered puberty, one must know the order of the appearance of secondary sexual characteristics. D. A growth spurt is usually not appreciated until at least 14 years of age for most boys. It is one of the last secondary sexual characteristics to be noted, following first ejaculations. E. First ejaculations usually occur around 13-14 years of age. These follow growth of the penis and scrotum, and precede a growth spurt.

John is a 17-year-old presenting today for a pre-participation physical exam. During the interview, he reports a low-grade fever, malaise, and headache for one week. In the past few days, his fever has gotten worse and he complains of a sore throat. He denies cough or chest pain. On physical examination, he is found to have a temperature of 101.3° F, and cervical lymphadenopathy and oropharyngeal erythema with exudate are noted. His participation would be most likely affected by which of the following tests? A. Chest x-ray B. CT head/neck C. EBV serologies D. Throat culture E. No further workup

6.1 > C has been selected by the expert. A. This choice is incorrect because there is no evidence of lower respiratory tract signs or symptoms. B. This choice is incorrect because the additional exposure to radiation is not warranted given the availability of other higher priority tests. C. This choice is correct because the patient's symptoms are suggestive of infectious mononucleosis. These include complaints of low-grade fever and malaise and findings of cervical lymphadenopathy and pharyngeal exudate. If testing is positive, the patient should be restricted from strenuous activity or contact sports during his illness due to the risk of splenic rupture. D. An antigen test (and culture, if negative) should be performed for streptococcal pharyngitis, but this clinical picture is more consistent with EBV, and mononucleosis is more likely to preclude participation in sports. After 24 hours of antimicrobials, the patient with streptococcal pharyngitis is no longer contagious. E. This choice is incorrect because the patient's symptoms and findings warrant further evaluation.

A 17-year-old boy presents for a sports pre-participation physical. He reports that he occasionally gets short of breath and feels light-headed with exercise, and sometimes he experiences chest pain as well. He lost consciousness once last season during a playoff basketball game, but attributed it to feeling sick at the time. His grandfather died suddenly at age 35 of unknown etiology. Which of the following is the most likely diagnosis? A. Hypoglycemia B. Congenital heart block C. Postural hypotension D. Prolonged QT syndrome E. Ventricular septal defect

6.2 > D has been selected by the expert. A. Hypoglycemia is very uncommon in healthy children. It is usually associated with diaphoresis, anxiety, tremulousness, and a feeling of hunger. Chest pain and shortness of breath are less likely. B. Congenital heart block can be caused by congenital heart defects and autoimmune disease in pregnant women (lupus). This condition is a rare cause of arrhythmia and is often diagnosed early in life. Many patients require a pacemaker. C. Postural hypotension is a very common cause of dizziness and visual changes in the pediatric population. It is usually triggered by volume depletion and skipping meals. Hypotension is not usually associated with shortness of breath, chest pain, and prolonged (> 5 min) loss of consciousness. D. Prolonged QT syndrome can cause syncopal episodes in late childhood or adolescence. QT intervals are elongated on ECG and lead to arrhythmias, like ventricular fibrillation. This condition is often associated with other abnormalities, including severe congenital sensorineural deafness. E. Ventricular septal defects often have negligible cardiovascular sequelae, or will close up on their own after a number of years. If they are not monitored, however, the shunting can cause hypertrophy of the right heart, called Eisenmeiger syndrome, which could result in pulmonary hypertension, exercise intolerance, and eventual heart failure. It would be unlikely, however, to cause chest pain or syncope. The family history of sudden cardiac death also points to prolonged QT rather than a VSD.

A 16-year-old boy presents to your office requesting clearance to play football. You begin by taking his medical history. He says that he feels very well, but admits that he recently experienced one episode of syncope that occurred when he trained really hard for football tryouts with his friends. He denies any shortness of breath, or chest pain currently. Family history is significant for an uncle who died of heat stroke at the age of 30 while playing basketball. Physical examination reveals no abnormalities. What is the next best step in management? A. ECG now, and if normal, reassurance B. Medically clear him to play C. Stress test D. ECG and referral to cardiology E. Observe and follow up in 6 months

6.3 > D has been selected by the expert. A. An ECG is indicated in all cases of syncope. Neurocardiogenic syncope, also known as vasovagal syncope, occurs when the brain is not being adequately perfused. Predisposing factors may include dehydration, hypovolemia, and standing up too quickly. It is not usually dangerous, and in these cases an ECG would be an appropriate first step. However, in cases of atypical syncope, as above, in which syncope occurs with exertion, and ECG would not be a sufficient work up. B. All medical issues must be resolved prior to clearing a patient to participate in a sport. Syncope with exertion, during exercise, is very concerning, and this patient should be worked up appropriately prior to clearance. C. A stress test is appropriate for evaluating chest pain, particularly in those at risk for atherosclerosis. However, in a patient with known syncope with exertion, a stress test might actually be dangerous. In addition, it only considers vessel disease as a source of pain, and does not take into account structural issues like an outlet obstruction. D. Referral to cardiology is the absolute next best step! The combination of syncope with exertion and a family history of a young death is concerning for something like hypertrophic cardiomyopathy. Don't be fooled about heat stroke. That is a positive family history for sudden death in a young person. This patient must be evaluated by cardiology, even if you don't hear a cardiac murmur! E. Observation is not appropriate here. As mentioned above, the syncopal event and positive family history are concerning. It would be inappropriate to just observe if this patient has hypertrophic cardiomyopathy, a very significant risk factor for sudden death. Cardiology consult is the only appropriate option here.

A 16-year-old previously healthy boy comes to the Pediatrics Urgent Care Clinic having "almost fainted" at soccer practice. He says that he had not eaten much earlier in the day and it was very hot and muggy outside. He felt light-headed and sick to his stomach. He denies losing consciousness and did not fall to the ground. He denies any chest pain. When you examine him, his eyes are sunken and he is tachycardic. What would be your next step in his management? A. Electrocardiogram (ECG) B. Measure his blood glucose C. Echocardiogram D. Give fluids and recheck his vital signs E. Stress test

6.4 > D has been selected by the expert. A. This choice is incorrect. While an EKG would be useful to assess for possible arrhythmia (long QT syndrome, WPW) or hypertrophic cardiomyopathy, the patient did not have a syncopal event or chest pain. As a result, an EKG would not be the most appropriate next step in diagnosis. B. This choice is incorrect. While the patient has had poor PO intake, hypoglycemia is relatively uncommon in an otherwise healthy young male. In addition, we would expect symptoms including diaphoresis, anxiety, and tremulousness, as opposed to dizziness. C. This choice is incorrect. Obstructive outlet pathology is less likely given the presentation, as no syncopal episode occurred. An echocardiogram may be indicated, but it would not be the next step in the evaluation. D. This choice is correct. The patient is likely dehydrated given the dizziness without loss of consciousness in the setting of poor PO intake, hot weather and exercise. As the symptoms occurred while he was upright, the likely mechanism is vasovagal. His sunken eyes and tachycardia are signs of moderate to severe dehydration. Since this is a clinical diagnosis, fluids should be given with subsequent rechecking of heart rate and blood pressure to confirm the diagnosis. E. This choice is incorrect. A cardiac etiology like hypertrophic cardiomyopathy is less likely given the presentation, and a stress test would not be the next step in the evaluation.

Claire is a 16-year-old girl who presents for birth control management. Her review of symptoms is unremarkable except for chest pain. When you ask her more questions, she reveals the pains are intermittent, on and off for the past couple months. It is not associated with exertion, sharp, and well localized at the left sternal border. It is very brief, lasting only a few seconds, during which she says she sometimes notices it gets worse when she breathes in. She denies recent URI or viral illness. The family history is negative for early cardiac disease. Her vital signs and physical exam are normal. Which is the next best step in management? A. ECG B. Reassurance C. Referral to a cardiologist D. Fast ultrasound of pericardial window E. Chest x-ray

6.5 > B has been selected by the expert. A. An ECG would be next if you suspected a cardiac abnormality. This would be more likely if the patient described something more like angina, a crushing chest pain or pressure, for longer periods of time rather than a few seconds, and aggravated by exercise. B. Based on the history, and assuming your physical exam is unremarkable, this sounds most suggestive of precordial catch syndrome, the most common cause of chest pain in an adolescent. No further workup is needed. C. Associated symptoms of syncope or palpitations would suggest a more severe cardiac abnormality and would prompt a referral to a cardiologist. The vignette states the rest of the ROS is negative and this would not warrant further evaluation now by a cardiologist. D. This would be the management if you suspected pericarditis or a pericardial effusion, but she denies recent URI or viral infection, and your PE is not notable for a pericardial friction rub suggestive of pericarditis. Her vitals are also stable and you might expect fever in pericarditis. E. A chest x-ray would be indicated if you suspected pneumonia that potentially might present with chest pain. However, the vignette didn't mention anything about cough and the patient is afebrile, making this less likely.

Adam is a 2-hour-old infant born at 32 weeks' gestational age via spontaneous vaginal delivery to a healthy mother with negative group B streptococcus status. There was no premature rupture of membranes and no meconium in the amniotic fluid. His Apgars were 8 at one minute and 9 at five minutes. Over the last two hours he has become progressively tachypneic. On physical examination he is large for gestational age. His vital signs are respiratory rate 75, temperature 36.5 C and heart rate is 130 beats per minute. His lung exam is remarkable for intercostal and subcostal retractions, grunting, and equal breath sounds. His heart exam reveals normal rhythm, normal S1 and S2, no murmurs, and normal peripheral pulses and capillary refill. Which of the following is the most likely cause of the infant's condition? A. Transient tachypnea of the newborn (TTN) B. Pneumothorax C. Congestive heart failure D. Respiratory distress syndrome E. Sepsis

7.1 > D has been selected by the expert. A. Transient tachypnea of the newborn (TTN) is much more common in infants born to diabetic mothers. TTN is unlikely because he is 32 weeks, very premature, and was born via NSVD. RDS is much more likely, although TTN is still a possibility and would need to evaluated with a CXR. B. Pneumothoraces are uncommon, but should always be considered in an infant with respiratory distress. Good bilateral air entry argues against this diagnosis. C. Congestive heart failure is an important cause of tachypnea. Adam has a normal cardiovascular exam, with no murmur, normal pulses, and good capillary refill. Infants with congestive heart failure usually present with the triad of tachypnea, tachycardia, and hepatomengaly. D. Respiratory distress syndrome (RDS) causes tachypnea and is therefore an important consideration in this case. RDS is more common in premature infants. Given the lack of history of maternal diabetes, an NSVD birth, and few risk factors for sepsis other than prematurity, Adam is likely to have RDS. E. Infants may present with respiratory distress from sepsis or from pneumonia. In this case, Baby Adam has a normal temperature, good blood pressure, and normal perfusion. While less likely, this diagnosis should always be considered in infants with respiratory distress.

A 3-hour-old infant, born by C-section at 36 weeks to a 30-year-old G1P1 with Apgars of 8 and 9 at 1 and 5 minutes, respectively, is found to be tachypneic in the newborn nursery. His mother has a history of Type II diabetes that was poorly controlled during her pregnancy. She was compliant with prenatal vitamins and took no other drugs during her pregnancy. Prenatal labs, including GBS, were negative. The mother's membranes ruptured 9 hours prior to delivery, she was afebrile, and the amniotic fluid had no meconium. On physical exam, the infant is large for gestational age. He has good air movement through the lungs bilaterally, without retractions or nasal flaring. He appears well perfused with normal cardiac exam. He is not in a flexed posture and has a weak suck reflex. A screening test at 3 hours of life reveals blood glucose of 39 mg/dL. What is the most likely diagnosis? A. Hypoglycemia B. Transposition of the great arteries C. Transient tachypnea of the newborn D. Neonatal sepsis E. Pneumothorax

7.2 > A has been selected by the expert. A. Hypoglycemia is a common presentation in an infant born to a diabetic mother with poor glucose control during her pregnancy. The increase in maternal serum glucose stimulates fetal pancreatic beta cells to increase insulin production, and this hyperinsulinemic state leads to hypoglycemia when the placental glucose supply is discontinued after delivery. At < 4 hours of life, a glucometer reading of < 25 mg/dL without symptoms or < 40 mg/dL with symptoms would require intervention to correct the hypoglycemic state. This infant has signs of hypotonia, with absence of flexed posture and weak suck, and a blood glucose reading of 39 mg/dL, making hypoglycemia the most likely diagnosis. B. Transposition of the great arteries is a congenital heart defect in which the aorta and pulmonary artery are switched, resulting in poorly oxygenated blood pumped into the systemic circulation. The infant with transposition is generally cyanotic and will be in respiratory distress. This congenital defect is usual accompanied by a VSD, and maternal diabetes is a risk factor. This infant appears well perfused on exam, and has no murmurs on cardiac exam, making transposition a less likely diagnosis. C. Transient tachypnea of the newborn (TTN) is a condition characterized by delayed clearance of amniotic fluid from the infant's lung following birth (persistent postnatal pulmonary edema) resulting in respiratory distress. Infants born by C-section and to diabetic mothers are at an increased risk of TTN. X-ray findings include "wet" appearing lungs with significant perihilar streaking, interstitial and alveolar fluid, and fluid in the pleural space and along the fissures. TTN generally resolves within 24 to 48 hours and is treated symptomatically. D. Neonatal sepsis is most commonly caused by GBS, Listeria and E. coli, transmitted from mother to baby. Additional risk factors include premature rupture of membranes (> 18 hours prior to delivery), preterm delivery, and chorioamnionitis. Infants may present with fever, trouble breathing, jaundice, and lethargy. Our infant is premature and tachypneic, but he is afebrile with normal Apgars and no evidence of altered level of alertness. Furthermore, mother was GBS negative, afebrile (no chorioamnionitis), with no premature rupture of membranes, making this diagnosis less likely. E. Pneumothorax is collapse of lung tissue secondary to air accumulation in the pleural space. Risk factors for pneumothorax in an infant include previous intubation or underlying lung disease (such as severe respiratory distress syndrome). Characteristic physical exam findings include asymmetric breath sounds or decrease in breath sounds on one side. This infant has good air movement in bilateral lung fields, making this diagnosis less likely.

A male infant weighing 3200 grams is born to a G1P1 female at 39 weeks' gestational age via planned C-section. Maternal PMH is unremarkable, and GBS status is unknown. Apgars are 7 and 8 at 1 and 5 minutes of life, respectively. The delivery is uncomplicated, and the infant initially appeared in good condition. However, one hour following delivery the infant develops increasing respiratory distress. Respiratory rate is assessed as 90 breaths/min. All other vital signs are within normal limits. On exam, the infant is acyanotic with rapid respirations and robust capillary refill. Chest x-ray shows bilateral lung fields with the appearance of "a radio-opaque line of fluid in the horizontal fissure of the right lung." No air bronchograms are noted. What is the most likely etiology of the infant's respiratory distress? A. Transient tachypnea of the newborn (TTN) B. Respiratory distress syndrome (RDS) C. Neonatal sepsis D. Meconium aspiration

7.3 > A has been selected by the expert. A. Transient tachypnea of the newborn (TTN) is the most likely underlying etiology. This condition is caused by residual fluid in the infant's lungs following delivery, and usually resolves within several days. It is more common in babies delivered via C-section, as the normal mechanical force of labor that helps expel fluid from the lungs is lacking. Babies with TTN and other forms of respiratory distress are often unable to nurse and require feeding via NG tube until respiratory status stabilizes. B. Respiratory distress syndrome (RDS) is less likely than TTN in this case. RDS is more common in premature infants and infants born to diabetic mothers. On chest x-ray, RDS is characterized by a ground-glass appearance and air bronchograms. C. Neonatal sepsis is possible, especially given the mother's unknown GBS status, but relatively unlikely compared to the other options, especially given the mode of delivery. Sepsis can certainly cause respiratory distress and, if suspected, should be promptly evaluated with screening labs and blood cultures. Neonatal sepsis is also more common with prolonged rupture of membranes (PROM) > 18 hours prior to delivery. D. Meconium aspiration can lead to respiratory distress, but seems less likely in this case given the infant's delivery via C-section. Additionally, meconium aspiration is more common when meconium is found in the amniotic fluid and/or products of conception. No mention of this was made in the above case description.

Adam is a newborn who was just born to a G2P1 mother at 36.2 weeks' gestation via a vaginal delivery. The mother reports that she did not receive prenatal care because she did not have insurance. She says that she thinks her "water broke" about two days ago, but she did not have any contractions after that, so she decided not to come to the hospital. She did not start having contractions until 19 hours before she delivered. After delivery, Adam did not cry vigorously, was tachypneic, cyanotic, and febrile to 100.5 F. Amniotic fluid did not contain meconium. His chest x-ray is normal. Given Adam's birth history, what is the most likely cause of his symptoms? A. Transient tachypnea of the newborn (TTN) B. Sepsis secondary to prolonged rupture of membranes C. Meconium aspiration syndrome D. Hypothermia E. Pneumothorax

7.4 > B has been selected by the expert. A. Transient tachypnea of the newborn (TTN) is a benign, self-limited condition caused by delayed clearance of lung fluid after birth. Patients with TTN usually have a classic chest x-ray that shows coarse fluffy densities that represent fluid-filled alveoli and/or fluid in the pleural space and a small amount of fluid in the fissures on the lateral view. Given Adam's normal chest x-ray and fever, it is unlikely that Adam has TTN. B. Prolonged rupture of membranes (PROM) is when the chorioamniotic membrane ruptures before the onset of labor. The main risks associated with PROM are preterm labor and delivery and neonatal sepsis. Adam's mom said that her "water broke" two days ago, which indicates that she had PROM. Adam's mother also did not receive prenatal care; therefore, she did not receive any of the prenatal screening tests that she should have, which increases the likelihood that she has an infection that could have potentially been transferred to Adam after the rupture of her membranes. Adam's history of PROM along with his fever and respiratory distress make this answer choice the best choice. C. Meconium aspiration syndrome occurs when the baby passes meconium in utero and aspirates the meconium either in utero or at the time of delivery. Since it was noted that the amniotic fluid did not contain meconium, it is less likely that Adam is suffering from meconium aspiration syndrome, even though he has symptoms of respiratory distress that can often be seen in meconium aspiration syndrome. D. Hypothermia can cause tachypnea of the newborn, especially premature newborns. However, hypothermia is less likely in Adam given his fever of 100.5 F. E. Pneumothorax is less likely in this case because of Adam's normal chest x-ray, but is important to consider in a tachypneic newborn. The presence of Adam's fever also makes pneumothorax less likely to be the sole cause of his symptoms.

A newborn is born at 30 5/7 weeks' gestation after induction of labor for the severe maternal preeclampsia. He is noted to have subcostal and intercostal retractions, grunting, nasal flaring, persistent cyanosis, and tachypnea 30 minutes after delivery. Apgars were 6 (-2 for color, -1 for breathing and -1 for tone) and 7 (-2 for color and -1 for breathing) at 1 and 5 minutes, respectively. Due to lack of prenatal care and the mother's presentation with severe preeclampsia, betamethasone x 1 was given during induction, but she did not receive a second dose prior to delivery. A chest x-ray is obtained, which reveals diffuse ground-glass appearance and air bronchograms bilaterally. What is the most likely diagnosis? A. Meconium aspiration syndrome (MAS) B. Respiratory distress syndrome (RDS) C. Persistent pulmonary hypertension (PPHN) D. Transient tachypnea of the newborn (TTN) E. Bronchopulmonary dysplasia (BPD)

7.5 > B has been selected by the expert. A. This choice is incorrect. Although the presence or absence of meconium was not noted in the case, it is known that from 20 to 34 weeks' gestation, the fetus will pass meconium infrequently. Most cases of MAS are in term or post-term infants. On chest x-ray, we might see overdistention of the lung or other sequelae, such as pneumothorax. B. This choice is correct. The baby boy is preterm, and his mother received only one dose of betamethasone, which puts him at increased risk for developing infant RDS, which is caused by insufficient surfactant. His physical exam and chest x-ray findings are consistent with RDS. C. This choice is incorrect. PPHN generally occurs in babies born after 34 weeks. There are several causes for PPHN: underdevelopment, maldevelopment, and maladaptation. Underdevelopment of the lungs can be secondary to congenital diaphragmatic hernia, oligohydramnios in utero, IUGR, or renal agenesis. The underdevelopment causes increased pulmonary vascular resistance and has a poor prognosis. Maldevelopment involves remodeling of pulmonary vasculature and is associated with post-term delivery and meconium aspiration syndrome. Maladaptation can be caused by infection with GBS. Vasoactive mediators are activated by bacterial phospholipids, causing an increase in pulmonary vascular resistance. D. This choice is incorrect. TTN is a disorder of delayed reabsorption of fluid in the newborn's lungs. Prematurity, delivery by C-section, being large or small for gestational age, or having a diabetic mother are all risks. In order to be diagnosed with TTN, the baby would need to show improvement within several hours. Although this is on the differential for the newborn baby's condition based on clinical presentation, a chest x-ray should have shown perihilar streaking and other evidence of interstitial fluid. E. This choice is incorrect, because BPD is the result of prolonged mechanical ventilation. Our patient is at risk for developing this syndrome if he requires intubation. Chest x-ray may show atelectasis, inflammation, or pulmonary edema. With severe disease, the chest x-ray may reveal fibrosis and hyperinflation.

A 4-day-old infant presents for his first pediatric well-child visit. His birth history consists of an uncomplicated normal spontaneous vaginal delivery after 7 hours of labor-no vacuum or forceps assistance were used. The patient is the first child to a 30-year-old mother of Mediterranean descent. Mom is very concerned that her baby has started to look "yellow" since leaving the hospital. She has been breastfeeding every 2-3 hours and says that the baby latches on for 1-5 minutes for each feed. He has had few wet diapers, and mom is concerned he is not getting enough to eat. Which of the following would most aid in narrowing the differential diagnoses? A. Newborn screen results B. Fractionated bilirubin C. WBC D. Blood smear E. No further workup is needed, as this is likely physiologic jaundice

8.1 > B has been selected by the expert. A. The newborn screen result normally takes a few weeks to process. However, even if the results were available, they might suggest two possibilities in the differential diagnosis of jaundice: hypothyroidism or G6PD deficiency. Even though these diagnoses are important to rule out, the test would not provide more information about the infant's current state of health or the next step in treatment. B. The test that will give you the most information at this juncture is a fractionated bilirubin. With the knowledge of the total serum bilirubin (TSB) and direct serum bilirubin, one will be able not only to narrow the differential (hemolysis vs. obstruction), but also to guide treatment (i.e., indirect serum bilirubin may be above phototherapy level). TSB can also indicate if the situation requires more drastic measures, such as a transfusion exchange. C. A WBC could be helpful if the infant is at risk or showing signs of infection or sepsis. The child did not have a complicated birth history, and now has no signs of fever or distress. Although it may be helpful to check a CBC and know the hemoglobin level, in case of hemolysis, a WBC will not help narrow the differential as much as a fractionated bilirubin given the above presentation. D. A blood smear would be helpful to diagnose hemolysis or RBC membranopathies. However, a blood smear would not be the most useful in narrowing the wide range of possibilities in the current differential, because it would help with identifying hemolytic causes. At this juncture, a fractionated bilirubin would be the most useful test to order first. E. Although physiologic jaundice is the most likely cause of jaundice at this age for an infant, there is not yet enough information to rule out a more serious cause. Because the mother is of Mediterranean descent, it is very possible that the cause of jaundice is G6PD deficiency, so further workup is definitely necessary. At the very least, a fractionated bilirubin is needed, and then likely a CBC.

A concerned mother brings her 7-day-old infant to your office after noticing yellowing of his skin for 2 days. She has also noticed he has not been gaining weight since she brought him home from the hospital 5 days ago. This is her first son and she has been trying to do everything perfectly, including breastfeeding him, since she was told that breast milk provides adequate nutrients and other healthy benefits, like antibodies and growth factors. However, upon further questioning, she is feeding him only 6 times a day for 10 minutes each time. She admits her breasts often feel full and are not relieved by nursing. He was born full term by spontaneous vaginal delivery but had a hard time sucking with breastfeeding. Upon exam, he looks dehydrated and appears to have jaundice of the face and chest. He has also lost > 10% of his birth weight. What could be the cause of his jaundice? A. Breast-milk jaundice B. Physiologic jaundice C. Sepsis D. Breastfeeding jaundice E. Crigler-Najjar syndrome

8.2 > D has been selected by the expert. A. Breast-milk jaundice is incorrect, because although it is a cause of jaundice associated with breastfeeding, it is NOT a result of low milk volume. In the above vignette, the infant does not appear to be breastfeeding well, which is multifactorial-poor suck and low number of feeds for the infant. Breast-milk jaundice would appear only if the infant were feeding well, although it is not completely understood what causes this form of jaundice. B. Physiologic jaundice is incorrect because this jaundice usually peaks at 3-4 days of life to full-term, healthy infants. This type of jaundice is not associated with breastfeeding but could be from numerous factors such as increased bilirubin production from short-lived RBCs or lack of intestinal flora to metabolize bile. No treatment is required for this type and it typically resolves on its own. C. Sepsis is incorrect because the infant does not clinically appear sick or toxic, and jaundice would be just one sign of a serious infection. Other symptoms may include respiratory distress, lethargy, poor feeding, vomiting, apnea, and temperature instability. Sepsis causes an elevated total and direct bilirubin and is highly unlikely when jaundice is the only symptom. Breastfeeding can have some protection against infection. D. Breastfeeding jaundice is the correct answer because it usually appears early in the first week of life and is caused by various factors, including poor breast milk intake. A decreased milk supply leads to limited enteral intake and can lead to increased enterohepatic circulation. Increased enterohepatic circulation describes the process where unconjugated bilirubin is reabsorbed in to the bloodstream where it binds to albumin and is recirculated. E. Crigler-Najjar syndrome is an incorrect choice because it appears within the first days of life and persists thereafter and is a relatively rare disease. This type of jaundice is caused by decreased bilirubin clearance from deficient or completely absent UDPGT (the enzyme that conjugates bilirubin with glucuronide to make it water-soluble and able to undergo excretion into bile).

A 5-day-old infant presents with a chief complaint of jaundice. As you obtain a careful history and physical examination, which of the following would NOT be a risk factor for jaundice in this infant? A. Mediterranean origin B. Prolonged labor with use of forceps during the delivery C. Mother is type O+ and baby is type B D. Phenylketonuria E. Poor breastfeeding during first few days of life

8.3 > D has been selected by the expert. A. Families of Mediterranean origin have higher risks of hereditary diseases that can cause jaundice. Glucose-6-phosphate dehydrogenase (G6PD) deficiency and the thalassemias are more common in Mediterranean families. Both cause hemolysis of RBCs, which results in an increased of jaundice. B. Difficult deliveries and birth trauma may result cephalohematomas, or hemorrhage of blood between the skull and the periosteum. These result from the rupture of blood vessels crossing the periosteum, usually caused by a prolonged second stage of labor or the use of forceps or other instrumentation during delivery. As the cephalohematoma resolves over weeks and the blood is reabsorbed, the breakdown of RBCs from the hematoma can result in increased bilirubin levels. C. ABO compatibility is a common cause of hemolysis in newborns. Since the mother has blood type O, she makes IgG antibodies to A-antigens and to B-antigens, leading to hemolysis of the fetal red blood cells. This can be detected by a positive direct Coombs test. D. Phenylketonuria (PKU) is an autosomal recessive metabolic disorder due to a mutation in phenylalanine hydroxylase, which is required to convert phenylalanine to tyrosine. PKU leads to buildup of phenylalanine in the brain, leading to mental retardation, seizures, and death if not detected and treated early. It is not associated with jaundice. E. Poor breastfeeding during the first few days of life and decreased enteral intake may result in breastfeeding jaundice. This may be caused by the mother who does not make enough milk, or by the infant with inadequate feeding or intake. Decreased intake leads to decreased motility of the GI tract and retention of meconium. The meconium contains β-glucuronidase, which hydrolyzes the conjugated bilirubin to an unconjugated form, which is reabsorbed and re-circulated into the blood through the enterohepatic circulation, increasing bilirubin levels in the blood.

A 3-week-old infant is brought to his pediatrician with a chief complaint of light tan-colored stools and worsening jaundice. He is exclusively breastfed and has 6-8 wet diapers per day. On exam, he appears to have scleral icterus and jaundice. Upon further workup, he is found to have an elevated direct bilirubin. What is his most likely diagnosis? A. Biliary atresia B. Breastfeeding jaundice C. G6PD deficiency D. Physiologic jaundice E. Caput succedaneum

8.4 > A has been selected by the expert. A. Biliary atresia can present anytime between birth and 8 weeks of age, but usually occurs after 2 weeks of age. Jaundice is usually the first presenting finding, along with acholic stools, dark urine (from increased bilirubin excretion) and hepatosplenomegaly if the problem goes unrecognized. Laboratory values classically show an increased level of direct or conjugated bilirubin > 2 mg/dL. If biliary atresia is confirmed with further laboratory testing and imaging, surgical intervention must be pursued as soon as possible. B. Breastfeeding jaundice normally occurs within the first week of life, most often because of decreased intake leading to dehydration and increased enterohepatic circulation. This patient is exclusively breastfed, but his jaundice began at approximately day 16 of life. Also, breastfeeding jaundice increases unconjugated bilirubin levels, making this answer choice less likely. C. G6PD is an X-linked inherited disorder. The severity of this disorder is dependent upon the degree of deficiency of the enzyme, but may present with neonatal unconjugated hyperbilirubinemia. However, laboratory findings show hemolytic anemia in the symptomatic state. Hemolysis is usually elicited by drugs, most notably primaquine and dapsone, as well as fava beans. D. Physiologic jaundice peaks at 3-4 days of life and generally resolves within a day or two. This patient is well beyond that age, making this answer less likely. E. Caput succedaneum is caused by an increase in serum above the periosteum of an infant that crosses suture lines (as opposed to cephalohematoma which does not cross suture lines). The increased serum bilirubin does not usually lead to significant hyperbilirubinemia.

A two-month-old infant is brought by her mother to clinic for a well-baby checkup. Mom says that her daughter is easy to care for because "she rarely cries and sleeps most of the time." On exam, the patient has a yellow tint to the skin, decreased muscle tone, and a large anterior fontanel. What is the most likely diagnosis in this patient? A. Sepsis B. Congenital adrenal hyperplasia C. Congenital hypothyroidism D. Shaken baby syndrome E. Neonatal lupus

9.1 > C has been selected by the expert. A. Sepsis is a blood infection of the infant. Signs and symptoms may include body temperature change, changes in respiration, increased or decreased heart rate, reduced movement, reduced feeding, low blood sugar, seizures, and jaundice. B. Infants with congenital adrenal hyperplasia often have abnormal genitalia (females), poor feeding, vomiting, dehydration, and electrolyte changes. C. Congenital hypothyroidism may not be clinically evident until 6 weeks of age due to circulating maternal thyroid hormone transmitted from the placenta. Signs and symptoms of congenital hypothyroidism include feeding problems, large fontanelles, hypotonia, large tongue, coarse cry, and frequently an umbilical hernia. Congenital hypothyroidism should be picked up on routine neonatal screening. D. Shaken baby syndrome may result in significant head trauma, including subdural hemorrhage, hypoxic/ischemic brain injury, and retinal hemorrhage. E. Neonatal lupus should be considered when the infant's mother has lupus. Cutaneous findings may be present at birth or may develop within the first 2 to 5 months of life. These findings include erythematous plaques, telangiectasias, or atrophic lesions. Affected infants may have cardiac abnormalities or conduction deficits. Hematologic disturbances may occur within the first 2 weeks of life.

A 2-week-old infant is brought by her mother to the clinic because of concerns for jaundice, constipation, sleepiness, and poor feeding. She has not had any vomiting. The patient was born at home and received no medications or lab studies. Physical exam reveals enlarged anterior fontanelle, jaundice, hypotonia, and an umbilical hernia. The remainder of the examination is normal. After confirmation of the diagnosis, which of the following is the most appropriate pharmacotherapy in this patient? A. Glucose and electrolyte supplementation B. Glucocorticoid and mineralocorticoid supplementation C. No treatment needed D. Consult with pediatric endocrinologist and start treatment with 10 to 15 mcg/kg/day of crushed levothyroxine in liquid, and follow up every 12 months E. Empiric antibiotics after collection of blood, urine, and CSF cultures

9.2 > D has been selected by the expert. A. This choice is incorrect because this is the treatment of choice for acutely ill children with dehydration, hypoglycemia, and perhaps infants with congenital adrenal hyperplasia, not congenital hypothyroidism as is most likely in this infant. B. This is the recommended treatment for corticoid and mineralocorticoid deficiency as seen in congenital adrenal hyperplasia. Clinical evaluation of this patient does not show virilization of this female infant, commonly seen in congenital adrenal hyperplasia. However, it would still be important for our patient in this case to undergo newborn screening, which includes screening for this disorder. C. This choice is incorrect because congenital hypothyroidism requires treatment within 2 weeks of onset of symptoms to mitigate severe brain damage and cognitive impairment. No treatment is needed for children born with hypothyroxemia of prematurity without TSH elevation. D. This choice is correct because the American Academy of Pediatrics recommends this treatment regimen for infants age 0 to 6 months old. Dosing is based upon age and weight. It would also be important to consult with a pediatric endocrinologist to evaluate the short and long-term treatment plan. E. This choice is incorrect. Although sepsis must always be considered in a neonate with jaundice, there is no indication of bacterial infection in this infant with other signs and symptoms of congenital hypothyroidism.

A 6-week-old infant is brought by her parents to the clinic for increased sleepiness for the past 2 weeks. The patient is not easily aroused for feedings and is not as alert as she was previously. The infant was born in Honduras and the family recently immigrated to the United States. Vital signs are normal. Physical exam reveals jaundice, an enlarged fontanelle and an umbilical hernia. Neonatal screening was not performed. Which of the following laboratory values would most likely be abnormal in this patient? A. Low sodium, high potassium B. Glucose < 40 mg/dL C. High WBC with bandemia D. High TSH, low T4

9.3 > D has been selected by the expert. A. A low sodium and high potassium in a lethargic infant would suggest the diagnosis of congenital adrenal hyperplasia (CAH), characterized by a decreased production of cortisol and aldosterone. Low aldosterone results in decreased stimulation of the H/K exchange in the collecting duct, hence loss of sodium, retention of potassium, and dehydration. In combination with low cortisol levels, patients in adrenal crisis may progress to shock, and death is not treated. The usual age of presentation is 1 to 2 weeks of age. Initial laboratory studies would include serum electrolytes, renin, cortisol and cortisol precursors, androgens, and glucose levels. Note that low cortisol will also impact gluconeogenesis and glycogenolysis. This diagnosis would be less likely in a non-viralized female. B. Symptoms of hypoglycemia in the neonate may include jitteriness, tremors, hypotonia, poor feeding and seizures. Management includes STAT glucose levels and intervention with parenteral or oral glucose, as indicated. The work up will include laboratory studies to rule out hyperinsulinism (IDM, insulinoma, prematurity), increased metabolic demand (polycythemia, sepsis, asphyxia), and inborn errors of metabolism (galactosemia, glycogen storage diseases, maple syrup urine disease). This patient did not present with jitteriness, tremors, or seizures. C. Sepsis should always be considered in lethargic neonates. While septic infants may present with fever or hypothermia, they may also be afebrile. This child did not appear to be acutely ill or toxic in appearance. D. Congenital hypothyroidism may present with poor feeding, constipation, jaundice (longer and more persistent than physiologic jaundice of newborn), mottled skin, large fontanelles, hypotonia, hypothermia and an umbilical hernia. Later findings include a hoarse cry, macroglossia, and myxedematous facies. Patients usually remain asymptomatic until after 6 weeks of age, as maternal thyroid hormones may still be in younger infants. Patients with primary hypothyroidism will have high TSH and low T4 levels. The most common cause of primary hypothyroidism will be aplasia or hypoplasia of the thyroid gland, and-much less commonly-inborn errors of metabolism. Secondary or tertiary hypothyroidism (HPA dysfunction) will have both low or inappropriately normal TSH and low T4, and are relatively rare causes of hypothyroidism in infants.

A 6-week-old infant is brought to the emergency room by his mother due to sleepiness, constipation, and yellow skin for the past 3 weeks. The mother and the baby recently immigrated to the United States from Malaysia. No medical records from the delivery are available. The infant has been breast-fed since birth. Vital signs are normal. Physical examination reveals jaundice, large anterior and posterior fontanelles, a large tongue, and abdominal distension. Which of the following is the most appropriate next step in management of this patient? A. Phototherapy B. Exchange transfusion C. Thyroid function studies D. Head ultrasound E. Family history

9.4 > C has been selected by the expert. A. Phototherapy is the treatment for physiologic jaundice, which peaks at three to four days and resolves by the fourth or fifth day of life. Lethargy, macroglossia, and umbilical hernia are not known to be caused by or associated with physiologic jaundice. This constellation of physical exam findings is more consistent with untreated congenital hypothyroidism. B. This choice is incorrect because this is the management of infants with hyperbilirubinemia approaching levels of concern for kernicterus. C. This choice is correct because the constellation of baby's problems is best accounted for by untreated congenital hypothyroidism. Unfortunately, severe mental retardation is unavoidable at this point because this condition should have been treated since birth. In the U.S., it would have been detected on the newborn screen. D. This choice is incorrect as there is no indication of hydrocephalus or concern for bleeding. E. This choice is incorrect because-although defects in biliary metabolism such as Gilbert's syndrome, seen in 5% of the population, can cause harmless jaundice-this patient has many other findings in addition to jaundice. Of the answer choices given, only congenital hypothyroidism fully accounts for the entire constellation of findings.

A 5-month-old is brought by her parents to the clinic because of decreasing oral intake over the past 4 days. The patient has been sleeping more than previously and seems to tire out when feeding. The patient breast feeds and eats home-made pureed vegetables. The patient has not had a bowel movement in 3 days. She has no fever or respiratory symptoms. Physical examination reveals a weak cry and decreased strength. What additional physical examination findings would be expected with this patient's presumptive diagnosis? A. Vesicular rash on her scalp B. Large tongue and umbilical hernia C. III/VI systolic murmur D. Absent deep tendon reflexes E. Cataracts and hepatosplenomegaly

9.5 > D has been selected by the expert. A. A vesicular rash may be seen in neonatal HSV infection, which can be a cause of encephalitis. It is less likely in this older age group and would most likely present with fever and possibly seizure. B. Although congenital hypothyroidism can present with lethargy, constipation, and poor feeding, the infant would be less likely to present with these symptoms as late as 5 months of age. C. Congenital heart disease may present with poor feeding, but a large VSD would likely present earlier and would not be associated with constipation and hypotonia. D. This infant likely has infant botulism which usually presents in the first year of life with hypotonia, lethargy, constipation, weak cry and can eventually lead to respiratory failure. These infants will have absent DTRs. E. An inborn error of metabolism can present with lethargy and poor feeding-and hepatosplenomegaly and eye findings may also be present-but this infant presented more acutely and at an older age than would be expected for a metabolic disorder.

A 3-day-old female is evaluated in the nursery for poor feeding since birth. The infant takes 1 ounce of formula every 4 hours. Each feed takes about 40 minutes because the patient falls asleep during the feed. The patient has vomited twice today and stooled once yesterday. Physical exam reveals a lethargic infant who is difficult to arouse by exam. The liver is enlarged and muscle tone is decreased. Serum ammonia level is elevated. Which of the following laboratory results would be expected in this patient? A. Polycythemia B. Elevated urine orotic acid C. Elevated 17-OH progesterone D. Elevated thyroid stimulating hormone E. Hyponatremia

9.6 > B has been selected by the expert. A. Although polycythemia can cause decreased feeding and decreased alertness, there should not be hyperammonemia. B. Elevated urine orotic acid is diagnostic of OTC deficiency, an x-linked condition, the most common urea cycle disorder. This diagnosis is most likely based on the enlarged liver, mental status changes and hyperammonemia. C. Elevated 17-OH progesterone would be expected and diagnostic of a patient with congenital adrenal hyperplasia (CAH) and would likely be associated with virilization in a female infant. Ammonia should not be elevated in CAH. D. Elevated TSH is diagnostic of congenital hypothyroism. This clinical scenario is not typical of congenital hypothyroidism. E. Infants with congenital adrenal hyperplasia may have low sodium, but not patients with OTC deficiency.


संबंधित स्टडी सेट्स

Field Tech III - IV Conventional-TCP/IP SERVICES (190E50-3)

View Set

3 Staying in Hotels (Top Notch 2)

View Set

Ch: 1 Accounting- Information for Decision Making

View Set

Exam 3 practice test Cultural Psychology

View Set

Male External Genitalia and Perineum

View Set